BLOCK 11: Mental Health Practice Questions

Ace your homework & exams now with Quizwiz!

ANS: D The behaviors mentioned are most consistent with criteria for CD, including aggression against people and animals; destruction of property; deceitfulness; rule violations; and impairment in social, academic, or occupational functioning. Intermittent explosive disorder is a pattern of behavioral outbursts characterized by an inability to control aggressive impulses in adults 18 years and older. The behaviors are not consistent with attention deficit or oppositional defiant disorder (ODD).

A 12-year-old has engaged in bullying for several years. The parents say, "We can't believe anything our child says." Recently this child shot a dog with a pellet gun and set fire to a neighbor's trash bin. The child's behaviors support which diagnosis? a. Attention-deficit/hyperactivity disorder (ADHD) b. Intermittent explosive disorder. c. Oppositional defiant disorder (ODD). d. Conduct disorder (CD)

ANS: A Some clients find that taking lithium with food diminishes nausea. The incorrect options are less helpful

A client diagnosed with bipolar disorder who takes lithium carbonate 300 mg three times daily reports nausea. To reduce the nausea most effectively, the nurse suggests that the lithium be taken with what? a. food. b. an antacid. c. an antiemetic. d. a large glass of juice.

ANS: C Systematic desensitization is a type of therapy aimed at extinguishing a specific behavior, such as the fear of flying. Psychoanalysis and short-term dynamic therapy seek to uncover conflicts. Aversion therapy involves use of a noxious stimulus, punishment, and avoidance.

A college student received an invitation to attend the wedding of a close friend who lives across the country. The student is afraid of flying. Which type of therapy would be most helpful for this client? a. Psychoanalysis b. Aversion therapy c. Systematic desensitization d. Short-term dynamic therapy

ANS: C Staying with a distraught client who is hearing voices serves several purposes: ongoing observation, the opportunity to provide reality orientation, a means of helping dismiss the voices, the opportunity of forestalling an action that would result in self-injury, and general support to reduce anxiety. Asking if the client hears voices is not particularly relevant at this point. Asking if the client plans to "get away from the voices" is relevant for assessment purposes but is less helpful than offering to stay with the client while encouraging a focus on their discussion. Suggesting playing cards with other clients shifts responsibility for intervention from the nurse to the client and other clients.

A newly admitted client diagnosed with schizophrenia says, "The voices are bothering me. They yell and tell me I am bad. I have got to get away from them." Select the nurse's most helpful reply. a. "Do you hear the voices often?" b. "Do you have a plan for getting away from the voices?" c. "I'll stay with you. Focus on what we are talking about, not the voices. " d. "Forget the voices and ask some other clients to play cards with you."

ANS: C Diaphoresis, weakness, and nausea are early signs of lithium toxicity. Problems mentioned in the incorrect options are unrelated to lithium therapy.

A nurse assesses a client who takes lithium. Which findings demonstrate evidence of complications? a. Pharyngitis, mydriasis, and dystonia b. Alopecia, purpura, and drowsiness c. Diaphoresis, weakness, and nausea d. Ascites, dyspnea, and edema

D. Exploring commonly held beliefs and values of the population

A nurse begins work in an agency that provides care to members of a minority ethnic population. The nurse will be better able to demonstrate cultural competence after... A. Identifying culture-bound issues B. Implementing scientifically proven interventions C. Correcting inferior health practices of the population D. Exploring commonly held beliefs and values of the population

b. Tell the nurse to stop discussing the behavior The greatest risk to this client is an invasion of privacy through the sharing of confidential information in a public place. The first action is to tell the new nurse to stop discussing this in a public place.

A nurse hears a newly licensed discussing a client's hallucinations in the hallway with another nurse. Which of the following actions should the nurse take? a. Notify the nurse manager b. Tell the nurse to stop discussing the behavior c. Provide an in service program about confidentiality d. Complete an incident report

B. Opiates The administration of methadone is indicated for a treatment of opiate use disorder. Opiates include opium, morphine, codeine, methadone, and heroin. Methadone is given as a substitute to prevent cravings and severe manifestation of opiate withdrawal.

A nurse in a substance use disorder treatment facilities reviewing the medication records of a group of clients. The nurse should expect to administer methadone for a client who has substance use disorder for which of the following substances? A. Amphetamines B. Opiates C. Barbiturates D. Hallucinogens

c. Attending a partial hospitalization program

A nurse in an acute mental health facility is assisting with a discharge planning for a client who has a severe mental illness and requires supervision. The client's partner works all day but is home by late afternoon. Which of the following strategies should nurse suggest for follow up care? a. Receiving daily care from a home health aide b. Having weekly visit from a nurse case worker c. Attending a partial hospitalization program d. Visiting a community mental health center on a daily basis

B. Hallucinations D. Diaphoresis E. Agitation Hallucinations, diaphoresis, and agitation are all indications of serotonin syndrome. Muscle tremors, rather than flaccidity, are an indication of serotonin syndrome. Fever, rather than hypothermia, is an indication of serotonin syndrome.

A nurse is assessing a client for hours after receiving an initial dose of fluoxetine. Which of the following findings should the nurse report to the provider as indications of serotonin syndrome? Select all that apply A. Hypothermia B. Hallucinations C. Muscular flaccidity D. Diaphoresis E. Agitation

D. Weight gain Weight gain is an expected adverse effect of paroxetine and other SSRIs. Other adverse effects include nausea, headaches, insomnia, and sexual dysfunction. The nurse should recognize that peripheral edema, chest congestion and shuffling gait are not adverse effects of paroxetine or other SSRIs.

A nurse is assessing a client has panic disorder and has been taking paroxetine. Which of the following assessments are the nurse identify as an adverse effect on this medication? A. Peripheral edema B. Chest congestion C. Shuffling gait D. Weight gain

B. Dizziness Dizziness is a common adverse effect of benzodiazepines. Other common adverse effects are drowsiness and sedation. Benzodiazepines are often prescribed for the treatment of seizure disorder. However, the sudden withdrawal of benzodiazepines can be associated with the development of seizures. Polyuria is an adverse effect of lithium, not benzodiazepines.

A nurse is assessing a client who has an anxiety disorder and is taking a benzodiazepine. For which of the following adverse effects should the nurse monitor the client for? A. Seizures B. Dizziness C. Polyuria D. Insomnia

a. Excessive worrying for 6 months d. Restlessness e. Sleep disturbances also would include procrastination in decision making and muscle tension so B & C are wrong

A nurse is assessing a client who has generalized anxiety disorder. Which of the following findings should the nurse expect? SATA a. Excessive worrying for 6 months b. Impulsive decision making c. Delayed reflexes d. Restlessness e. Sleep disturbances

D. Temperature Antipsychotic medication such as clozapine can cause agranulocytosis, which is the depletion of wbc's. This increases the clients risk for infection. A fever is an early indication to check the clients WBC count to detect agranulocytosis. antipsychotic medication do not typically affect the fluid balance, although they can cause urinary retention. Clozapine causes urinary retention. Closet pain is unlikely to cause hypertension, however, it can cause orthostatic hypotension.

A nurse is assessing a client who is receiving clozapine to treat schizophrenia. The nurse should identify an increase in which of the following parameters as an early indication of an adverse effect of this medication? A. Urine specific gravity B. Urine output C. Blood pressure D. Temperature

D. Akathisia Akathisia is an extrapyramidal adverse effect characterized by a sense of inner restlessness and observable behaviors such as pacing, rocking forward and backward in a chair and constant foot tapping. Dystocia is an extrapyramidal adverse effect characterized by muscle spasms, not motor restlessness. Parkinsonism is characterized by by shuffling gait, drooling and stooped posture similar to manifestations you see in Parkinson's disease. Tardive dyskinesia is an irreversible finding characterized by involuntary movements of the extremities.

A nurse is assessing a client with a psychotic disorder has a new prescription for haloperidol. The client is pacing in the hallways and states I can't seem to sit still which of the following extrapyramidal side effects is this client likely experiencing? A. Dystocia B. Parkinsonism C. Tardive dyskinesia D. Akathisia

c. Gradually expose the client to an elevator while practicing relaxation techniques

A nurse is assisting with systematic desensitization for a client who has an extreme fear of elevators. Which of the following actions should the nurse implement with this form of therapy? a. Demonstrate riding in an elevator, and then ask the client to imitate the behavior b. Advise the client to say "Stop" out loud every time they begin to feel an anxiety response related to an elevator c. Gradually expose the client to an elevator while practicing relaxation techniques d. Stay with the client in an elevator until the anxiety response diminishes

B. Drowsiness Drowsiness can be an adverse effect of this medication in a manifestation of serotonin syndrome. The nurse should notify the provider about the signing immediately. Diarrhea is an adverse effect of this medication not constipation. Bruising would be seen with NSAIDs and a cough would most likely be seen with an ace inhibitor.

A nurse is caring for a client who is taking selegiline. The nurse should monitor the client for which of the following adverse effects of selegiline and notify the provider if it occurs? A. Bruising B. Drowsiness C. Coughing D. Constipation

c. "I will need to discontinue this medication slowly." When discontinuing fluoxetine, the client should taper the med slowly according to prescribed dosing schedule to reduce risk of withdrawal syndrome

A nurse is caring for a client who is to begin taking fluoxetine for treatment of panic disorder. Which of the following statements indicates the client understands the use of this medication? a. "I will take the medication at bedtime." b. "I will follow a low sodium diet while taking this medication." c. "I will need to discontinue this medication slowly." d. "I will be at risk for weight loss with long term use of this medication."

b. Denial pretending the truth is not reality ch 4 ati

A nurse is caring for a client who smokes and has lung cancer. The client reports, "I'm coughing because O have that cold that everyone has been getting." The nurse should identify that the client is using which of the following defense mechanisms? a. Reaction formation b. Denial c. Displacement d. Sublimation

A. The client reports that hallucinations occur less frequently The nurse should identify that chlorpromazine, when used to treat schizophrenia, reduces elucidations. Chlorpromazine is a first generation conventional antipsychotic medication and is effective in decreasing delusions, hallucinations, and agitation. He can also treat manic behavior and client to have bipolar disorder.

A nurse is caring for a client with schizophrenia who has been taking chlorpromazine for the past two months. Which of the following findings demonstrate that the chlorpromazine has been effective? A. The client reports that hallucinations occur less frequently B. The client sleeps uninterrupted for six hours each night C. The client reports that she is the most important person on the unit D. The client demonstrates stereotyped behaviors

d. The client cannot recall anything that happened during the past 2 weeks The client with ASD often expresses dissociative manifestations regarding the event which includes a sense of unreality

A nurse is collection an admission history for a client who has acute stress disorder (ASD). which of the following client behaviors should the nurse expect? a. The client remembers many details about the traumatic incident b. The client expresses heightened elation about what is happening c. The client remembers first noticing manifestations of the disorder 6 weeks after the traumatic incident occurred d. The client cannot recall anything that happened during the past 2 weeks

a. Offering advice

A nurse is communicating with a client who was admitted for treatment of a substance use disorder. Which of the following communication techniques should the nurse identify as a barrier to therapeutic communication? a. Offering advice b. Reflecting c. Listening attentively d. Giving information

A. Phenelzine Phenelzine is a monoamine oxidase inhibitor that is prescribed for depression and other mental health disorders. I verse effects of phenelzine is orthostatic hypotension. The nurse should inform the client who is taking phenelzine that dizziness and lightheadedness are indications of hypotension. The nurse should also instruct the client to rise slowly from a lying or sitting position to minimize the drop in blood pressure.

A nurse is conducting a risk assessment for clients who are prescribed medications that can cause orthostatic hypotension. Which of the following medication's require a follow up by the nurse? A. Phenelzine B. Escitalopram oxalate C. Galantamine D. Naltrexone

b. Difficulty sleeping an indication of relapse c. Begin taking your medication as soon as relapse begins e. Anhedonia (the inability to feel pleasure) is a clinical manifestation of a depressive relapse

A nurse is discussing relapse prevention with a client who has bipolar disorder. Which of the following information should the nurse include in the teaching? Select all that apply a. Use caffeine in moderation to prevent relapse b. Difficulty sleeping an indication of relapse c. Begin taking your medication as soon as relapse begins d. Participating in psychotherapy can help prevent a relapse e. Anhedonia is a clinical manifestation of a depressive relapse

B. While taking this medication, I should keep a pack of sugarless gum Clonidine commonly causes clients to experience dry mouth. Chewing sugarless gum is an effective method to address this adverse effect. Clonidine is useful during opioid withdrawal. However it does not reduce cravings. Clonidine reduces, rather than causes diarrhea. Buprenorphine, rather than clonidine is administered sublingually.

A nurse is evaluating a clients understanding of a new prescription of clonidine for the treatment of opioid use disorder. Which of the following statements by the client indicates an understanding of the teaching? A. Taking this medication will help reduce my craving for heroin B. While taking this medication, I should keep a pack of sugarless gum C. I can expect some diarrhea from taking this medication D. Each dose of the medication should be placed under my tongue to dissolve

B. Opiates Administration of methadone is indicated for the treatment of opiate use disorder. Opiates include opium, morphine, codeine, methadone, and heroin. Methadone is used as a substitute to prevent cravings and severe manifestations of opiate withdrawal. The administration of methadone is not indicated in the other meds.

A nurse is in a substance use disorder treatment facility and reviewing the medication records of a group of clients. The nurse should expect to administer methadone for a client who has a substance use disorder for which of the following substances? A. Amphetamines B. Opiates C. Barbiturates D. Hallucinogens

B. Offer concise explanations C. Establish consistent limits E. Use a firm approach with communication

A nurse is planning care for a client who has bipolar disorder and is experiencing a manic episode. Which of the following interventions should the nurse include in the plan of care? Select all that apply A. Provide flexible client behavior expectations B. Offer concise explanations C. Establish consistent limits D. Disregard client concerns E. Use a firm approach with communication

C. Drinking caffeinated beverages will decrease the effectiveness of this medication The nurse should inform the client that consuming caffeine while taking benzodiazepines such as lorazepam will result in decreased effectiveness of the medication. Caffeine is a stimulant, and lorazepam is a CNS depressant therefore these substances will counteract each other. The client should avoid caffeine while taking this medication. Anti-acids interact with the benzodiazepine by dialing absorption. Benzodiazepines are CNS depressants and are expected to cause drowsiness, not insomnia. The client should notify the provider if the medication's effectiveness decreases. The client should not increase the dosage without a prescription.

A nurse is providing teaching for a client who has generalized anxiety disorder and a new prescription for lorazepam. Which of the following statements should the nurse include? A. Taking an anti-acid with this medication will decrease stomach upset B. Expect the medication to cause insomnia for the first one to two weeks C. Drinking caffeinated beverages will decrease the effectiveness of this medication D. Increase the dosage of the effectiveness of his medication decreases

B. Stop taking the herbal supplement while taking this medication Taking the antidepressant sertraline and the herbal supplement St. John's wort increases the clients risk for serotonin syndrome.

A nurse is providing teaching to a client has a new prescription for sertraline. The client asked the nurse if he should continue to take St. John's wort for depression. Which of the following instruction should the nurse give the client? A. Take the medication and herbal supplement together B. Stop taking the herbal supplement while taking this medication C. Take the herbal supplement and the medication at least 2 hours apart D. Take an antacid with both of herbal supplement and the medication

A. You should have your white blood cell count checked once per week for six months The nurse should instruct the client to have lab testing of wbc's and neutrophils every week for six months. Clozapine causes weight gain not weight loss. It also causes constipation.

A nurse is providing teaching to a client has a new prescription of clozapine. Which of the following statements should the nurse include in the teaching? A. You should have your white blood cell count checked once per week for six months B. You should check yourself every 3 days for weight loss C. You might experience frequent loose stools D. You might experience ringing in your ears

D. This medication should not be abruptly stopped The nurse should instruct the client that stopping venlafaxine abruptly will lead to manifestations of withdrawal. The nurse should instruct the client that venlafaxine is approved for both short-term or long-term use. However, dental Faxon is an anti-depressant that takes several weeks to reach its peak affect. Therefore, the medication cannot be used as PRN. The nurse should instruct the client that venlafaxine is effective against cognitive and psychic manifestations of anxiety but does little to decrease somatic manifestations.

A nurse is providing teaching to a client has generalized anxiety disorder and a new prescription for a venlafaxine. Which of the following statements to the nurse may? A. This medication is only for short term used to B. This medication can be taken on an as needed basis C. This medication will effectively reduce your physical manifestations of anxiety D. This medication should not be abruptly stopped

B. Pure vanilla extract The nurse should instruct the client to avoid alcohol and alcohol containing substances such as pure vanilla extract while taking disulfiram. Ingestion of alcohol while taking this medication causes hyperventilation, dizziness, vomiting and hypotension.

A nurse is providing teaching to a client that has a new prescription for a disulfiram for the management of alcohol dependence. Which of the following dietary items to the nurse instruct the client to avoid? A. Peppermint candy B. Pure vanilla extract C. Salt D. Chocolate

A. This medication will help prevent seizures during an alcohol withdrawal Carbamazepine is used during withdrawal to decrease the risk for seizures. It is also used to promote safe withdrawal rather than to decrease cravings for alcohol. Clonidine or propanolol is used during withdrawal to depress the autonomic response and its affect on blood pressure.

A nurse is providing teaching to a client who has alcohol use disorder and a new prescription for carbamazepine. Which of the following information should the nurse include in the teaching? A. This medication will help prevent seizures during an alcohol withdrawal B. Taking this medication will decrease your cravings for alcohol C. This medication maintains your blood pressure at a normal level during alcohol withdrawal D. Taking this medication will improve your ability to maintain abstinence from alcohol

C. Dizziness The nurse should inform the client that dizziness is a common adverse effect of buspirone. The nurse should instruct the client to avoid driving an operating heavy machinery until the presence of adverse effect has been determined. Confusion is not in an inverse effect of buspirone, although the client might experience decrease concentration and headaches. Tachycardia and palpations are possible adverse effects of buspirone. Drowsiness not insomnia, is an adverse effect of buspirone.

A nurse is providing teaching to a client who has generalized anxiety disorder and a new prescription for buspirone. Which of the following manifestations is a common adverse effect of this medication? A. Confusion B. Bradycardia C. Dizziness D. Insomnia

b. Yellowing skin d. Fever e. Malaise This medication can cause hepatotoxicity and give the client flu like symptoms, yellowing skin and abdominal pain. And this medication will also cause insomnia rather than somnolence and it will decrease the appetite rather than increase it.

A nurse is providing teaching to an adolescent client who is to begin a new prescription of atomoxetine for ADHD. The nurse should instruct the client to monitor for which of the following adverse effects? Select all that apply a. Somnolence b. Yellowing skin c. Increased appetite d. Fever e. Malaise

a. Seizures b. Agitation e. Irregular pulse Symptoms of a TCA toxicity include dysrhythmias, mental confusion, agitation, seizures followed by coma or possible death. Photophobia and dry mouth are anticholinergic effects, not signs of TCA toxcity.

A nurse is teaching the guardian of a child who has autism spectrum disorder about the indications of imipramine toxicity. Which of the following should the nurse include in the teaching? Select all that apply a. Seizures b. Agitation c. Photophobia d. Dry mouth e. Irregular pulse

ANS: B, D Individuals with antisocial personality disorders characteristically demonstrate manipulative, exploitative, aggressive, callous, and guilt-instilling behaviors. Individuals with antisocial personality disorders are more extroverted than reclusive, rarely show anxiety, and rarely demonstrate clinging or dependent behaviors. Individuals with antisocial personality disorders are more likely to be impulsive than to be perfectionists.

A nurse plans care for an individual diagnosed with antisocial personality disorder. Which characteristic behaviors will the nurse expect? (Select all that apply.) a. Reclusive behavior b. Callous attitude c. Perfectionism d. Aggression e. Clinginess f. Anxiety

ANS: d. confers with a pharmacist when selecting over-the-counter medications. Over-the-counter medicines may contain vasopressor agents or tyramine, a substance that must be avoided when the client takes MAOI antidepressants. Medications for colds, allergies, or congestion or any preparation that contains ephedrine or phenylpropanolamine may precipitate a hypertensive crisis. MAOI antidepressant therapy is unrelated to the need for sodium limitation, support stockings, or leg elevation. MAOIs interact with tyramine-containing foods, not selenium, to produce dangerously high blood pressure.

A nurse provided medication education for a client diagnosed with major depressive disorder who began a new prescription for phenelzine. Which behavior indicates effective learning? The client a. monitors sodium intake and weight daily. b. wears support stockings and elevates the legs when sitting. c. can identify foods with high selenium content that should be avoided. d. confers with a pharmacist when selecting over-the-counter medications.

ANS: D The client is likely laughing in response to inner stimuli, such as hallucinations or fantasy. Focus on the hallucinatory clue (the client's laughter) and then elicit the client's observation. The incorrect options are less useful in eliciting a response: no joke may be involved, "why" questions are difficult to answer, and the client is probably not focusing on what the nurse said in the first place.

A nurse sits with a client diagnosed with schizophrenia. The client starts to laugh uncontrollably, although the nurse has not said anything funny. What is the nurse's most therapeutic response? a. "Why are you laughing?" b. "Please share the joke with me." c. "I don't think I said anything funny." d. "You're laughing. Tell me what's happening."

a. Difficulty concentrating on tasks c. Negative self image d. Recurring nightmares Clients with PTSD are usually reluctant to talk about it and have increased startle reflex and hypervigilance

A nurse working on an acute mental health unit is caring for a client who has posttraumatic stress disorder (PTSD). Which of the following findings should the nurse expect? Select all that apply a. Difficulty concentrating on tasks b. Obsessive need to talk about the traumatic event c. Negative self image d. Recurring nightmares e. Diminished reflexes

A. "Sometimes I do stupid things."

A patient repeatedly stated, "I am stupid." Which statement by the patient would show progress resulting from cognitive behavioral therapy? A. "Sometimes I do stupid things." B. "Things always go wrong for me." C. "I always fail when I try new things." D. "I'm disappointed in my lack of ability."

ANS: D Sertraline is an SSRI antidepressant medication, which suppresses REM sleep. Dreams would decrease because they occur during REM. Benzodiazepines reduce slow-wave sleep. SSRIs have a side effect of insomnia.

A person is prescribed sertraline 100 mg PO daily. Which change in sleep is likely secondary to this medication? a. More dreams. b. Excessive sleepiness. c. Less slow-wave sleep. d. Less rapid eye movement (REM) sleep.

ANS: A Reaction formation is an unconscious mechanism that keeps unacceptable feelings out of awareness by using the opposite behavior. Instead of expressing hatred for the other person, the individual gives praise. Denial operates unconsciously to allow an anxiety-producing idea, feeling, or situation to be ignored. Projection involves unconsciously disowning an unacceptable idea, feeling, or behavior by attributing it to another. Repression involves unconsciously placing an idea, feeling, or event out of awareness.

A person speaking about a rival for a significant other's affection says in an emotional, syrupy voice, "What a lovely person. That's someone I simply adore." The individual is demonstrating which defense mechanism? a. reaction formation. b. repression. c. projection. d. denial.

ANS: C Safety and physiological needs have the highest priority. Hyperactivity, poor nutrition, hydration, and not sleeping take priority in terms of the needs listed above because they threaten the physical integrity of the client. The other behaviors are less threatening to the client's life.

A person was directing traffic on a busy street, rapidly shouting, "To work, you jerk, for perks" and making obscene gestures at cars. The person has not slept or eaten for 3 days. Which assessment findings will have priority concern for this client's plan of care? a. Insulting, aggressive behavior b. Pressured speech and grandiosity c. Hyperactivity; not eating and sleeping d. Poor concentration and decision making

ANS: C Hyperactivity (activity without sleep) and poor judgment (posting rhymes on government websites) are characteristic of manic episodes. The distracters do not specifically apply to mania.

A person was online continuously for over 24 hours, posting rhymes on official government websites and inviting politicians to join social networks. The person has not slept or eaten for 3 days. What features of mania are evident? a. Increased muscle tension and anxiety b. Vegetative signs and poor grooming c. Poor judgment and hyperactivity d. Cognitive deficits and paranoia

ANS: D The correct response uses exploring, a therapeutic technique. The distracters give advice, a nontherapeutic technique.

A school age child tells the school nurse, "Other kids call me mean names and will not sit with me at lunch. Nobody likes me." what is the nurse's most therapeutic response? a. "Just ignore them and they will leave you alone." b. "You should make friends with other children." c. "Call them names if they do that to you." d. "Tell me more about how you feel."

ANS: D PTSD precipitates changes that can lead to divorce. It is important to provide support to both the veteran and spouse. Confrontation will not be effective. While it is important to provide information, on-going support will be more effective.

A soldier returned home last year after deployment to a war zone. The soldier's spouse complains, "We were going to start a family, but now he won't talk about it. He will not look at children. I wonder if we're going to make it as a couple." Select the nurse's best response. a. "Posttraumatic stress disorder (PTSD) often changes a person's sexual functioning." b. "I encourage you to continue to participate in social activities where children are present." c. "Have you talked with your spouse about these reactions? Sometimes we just need to confront behavior." d. "Posttraumatic stress disorder often strains relationships. Here are some community resources for help and support."

B. This medication must be discontinued by a gradual tapering overtime Rapid withdrawal from lorazepam has been associated with manifestations of withdrawal such as anxiety, sleeplessness, and irritability. It is discontinued by gradually tapering overtime to avoid any adverse responses. Lorazepam is contradicted for use during pregnancy. The nurse should inform the client not to increase the dosage without consulting the provider. Lorazepam has no known effect on blood glucose levels, therefore, monitoring is not necessary while taking his medication.

Address is providing discharge teaching for a female client with anxiety disorder who has a new prescription for lorazepam. Which of the following instruction should the nurse include in the teaching? A. This medication can be safely taken during pregnancy B. This medication must be discontinued by a gradual tapering overtime C. An extra dose of this medication can be taken at bedtime if you experience insomnia D. You should monitor your blood glucose levels closely while taking this medication

ANS: B Selective serotonin reuptake inhibitor antidepressants are very safe in overdosage situations, which is not true of the other medications listed. Given this client's history of overdosing, it is important that the medication be as safe as possible in the event of another overdose of prescribed medication.

An adult outpatient client diagnosed with major depressive disorder has a history of several suicide attempts by overdose. Given this client's history and diagnosis, which antidepressant medication would the nurse expect to be prescribed? a. Amitriptyline b. Fluoxetine c. Desipramine d. Tranylcypromine sulfate

ANS: D The client's threat to kill self or others with the knife he possessed constituted a clear and present danger to self and others. The distracters are not sufficient reasons for seclusion.

An emergency code was called after a client pulled a dinner knife from a pocket and threatened, "I will kill anyone who tries to get near me." The client was safely disarmed and placed in seclusion. What is the justification for this use of seclusion? a. The client was threatening to others. b. The client was experiencing psychosis. c. The client presented an undeniable escape risk. d. The client presented a clear and present danger to others.

B. Nausea, diarrhea, and confusion Therapeutic range is 0.6-1.2; over 1.5 is toxic range, the beginning signs of toxicity is nausea, diarrhea and confusion

Client with bipolar disorder has been taking lithium and today the clients serum lithium level is 1.8 mEq/L. What effects would the nurse expect to see? A. Constipation and postural hypotension B. Nausea, diarrhea, and confusion C. Fever, muscle rigidity, and disorientation D. None the serum level is in the therapeutic range

ANS: C The three drugs in the stem of the question are all anticonvulsants. Lamotrigine is also an anticonvulsant. Clonazepam is an anxiolytic; aripiprazole and risperidone are antipsychotic drugs.

Consider these three anticonvulsant medications: divalproex, carbamazepine, and gabapentin. Which medication also belongs to this classification? a. clonazepam b. risperidone c. lamotrigine d. aripiprazole

ANS: C Clients manipulate and control staff in various ways. By keeping staff off balance or fighting among themselves, the person with an antisocial personality disorder is left to operate as he or she pleases. Seductive behavior has sexual connotations. The client is displaying the opposite of detached behavior. Guilt is not evident in the comments.

Consider this comment to three different nurses by a client diagnosed with an antisocial personality disorder, "Another nurse said you don't do your job right." Collectively, these interactions can be documented using which term? a. seductive. b. detached. c. manipulative. d. guilt-producing.

A. Orthostatic hypotension Orthostatic hypotension is an adverse effect of chloropromazine. Other adverse effects include palpitations, tachycardia, constipation, sedation, and photo sensitivity. Many of the adverse effects are due to anti-cholinergic actions.

I nurse is caring for a client who has schizophrenia and a prescription for chlorpromazine. For which of the following adverse effects do the nurse monitor? A. Orthostatic hypotension B. Diarrhea C. Urinary frequency D. Bradycardia

A. Report of pain The nurse identify that naloxone is used to reverse effects of an opioid overdose administer for pain, sedation euphoria, and respiratory depression. Excess doses of an oxen can cause the return of pain but can improve the clients respiratory rate.

I nurse is caring for a client who received naloxone for a suspected opioid overdose. Which of the following findings should the nurse identify as an adverse effect of this medication? A. Report of pain B. Respiratory rate of eight per minute C. Report of numbness D. Report of abdominal cramping and diarrhea

D. Disulfiram Disulfiram is a type of aversion therapy that helps clients abstain from alcohol. Drinking alcohol while taking this medication produces a toxic reaction that causes vomiting, confusion, headaches, breathing difficulties, and other manifestations. Varenicline reduces nicotine cravings. Clonidine treats heroin withdrawal. Buprenorphine treats opioid withdrawal.

Nurse is caring for a client with alcohol use disorder who has undergone detoxification. Which of the following medication should the nurse expect the provider to prescribe to assist the client with maintaining sobriety? A. Varenicline B. Clonidine C. Buprenorphine D. Disulfiram

ANS: C Overly controlled eating behaviors, extreme weight loss, preoccupation with food, and wearing several layers of loose clothing to appear larger are part of the clinical picture of an individual with anorexia nervosa. The individual with bulimia usually is near normal weight. The binge eater is often overweight. The client with eating disorder not otherwise specified may be obese

Over the past year, a client has cooked gourmet meals for the family but eats only tiny servings. This person wears layered loose clothing and currently weighs 95 pounds, after a loss of 35 pounds. Which medical diagnosis is most likely? a. Binge eating b. Bulimia nervosa c. Anorexia nervosa d. Eating disorder not otherwise specified

B. Poor physical health can lead to mental distress and disorders D. There is a strong relationship between physical health and mental health

The World Health Organization describes health as "a state of complete physical, mental and social well being and not merely the absence of disease or infirmity." Which statement is true in regard to overall health? SATA A. There is no relationship between physical and mental health B. Poor physical health can lead to mental distress and disorders C. Poor mental health does not lead to physical illness D. There is a strong relationship between physical health and mental health E. Mental health needs take precedence over physical health needs

a. Auditory hallucinations c. Delusions of grandeur d. Severe agitation First-generation antipsychotics treat positive symptoms of schizophrenia. positive symptoms include hallucinations, delusions, alterations in speech, bizarre behavior such as walking backwards constantly. Negative symptoms include flat or usually blunted affect, Alogia or poverty of thought or speech.Anergia, or a lack of energy. Anhedonia or a lack of pleasure or joy, and abolition or lack of motivation. Think of positive as over exaggerated or more of and negative as taking away from them.

The charge nurse is discussing manifestations of schizophrenia with a newly licensed nurse. Which of the following manifestations to the charge nurse identify as being affectively treated by first-generation antipsychotics? Select all that apply a. Auditory hallucinations b. Withdrawal from social situations c. Delusions of grandeur d. Severe agitation e. Anhedonia

ANS: C Negative symptoms include apathy, anhedonia, poor social functioning, and poverty of thought. Poor personal hygiene is an example of poor social functioning. The distractors are positive symptoms of schizophrenia. See relationship to audience response question

The nurse assesses a client diagnosed with schizophrenia. Which assessment finding would the nurse regard as a negative symptom of schizophrenia? a. Auditory hallucinations b. Delusions of grandeur c. Poor personal hygiene d. Psychomotor agitation

D. Providing services for mentally I'll individuals who require intensive treatment and continue to live in the community

The nurse assigned to an assertive community treatment ACT team explained to a family member of a client with schizophrenia that the programs treatment goal is: A. Assisting patients to maintain abstinence from alcohol and other substances of abuse B. Providing a daily outpatient program that is highly structured and therapeutic for patients with a strong support system C. Maintaining medications and stable psychiatric status for incarcerated inmates who have a history of mental illness D. Providing services for mentally I'll individuals who require intensive treatment and continue to live in the community

ANS: D Individuals with schizotypal personality disorder do not want to be involved in relationships. They are shy and introverted, speak little, and prefer fantasy and daydreaming to being involved with real people. The other behaviors would characteristically be noted in narcissistic, histrionic, and antisocial personality disorder.

The nurse caring for an individual demonstrating symptoms of schizotypal personality disorder would expect assessment findings to include what characteristics? a. arrogant, grandiose, and a sense of self-importance. b. attention seeking, melodramatic, and flirtatious. c. impulsive, restless, socially aggressive behavior. d. socially anxious, rambling stories, peculiar ideas.

C. Following established hospital protocols, report suspected abuse

The nurse is caring for a six-year-old child and discovers several injuries indicative of physical abuse. Which action by the nurse is of highest priority? A. Document location of each would in the electronic medical record B. Provide the child with contact information for self-defense course C. Following established hospital protocols, report suspected abuse D. Confront the parents of the child and determine who is abusing the child

B. Have you seen something that you thought wasn't real? Keyword perceptual

The nurse is conducting a mental status examination on a new patient. Which question would be most beneficial in assessing perceptual disturbances in the patient. A. How would you describe your experience as a child? B. Have you seen something that you thought wasn't real? C. Have you ever felt sad at a happy event, or happy outside event? D. What does it mean when someone says a dime a dozen?

D. Monitor lithium blood levels

The nurse is developing a teaching plan for a client taking lithium what would the nurse include? A. Avoid eating aged cheese, processed meats, and red wine B. Drink twice the usual daily amount of fluid C. Double the lithium dose if diarrhea or vomiting occurs D. Monitor lithium blood levels

C. Confusion

The nurse is developing a teaching plan for a client who has a prescription for fluoxetine. Which manifestation should the nurse report to the provider immediately? A. Insomnia B. Vomiting C. Confusion D. Fatigue

ANS: A Normal range for a blood sample taken 8 to 12 hours after the last dose of lithium is 0.6 to 1.2 mEq/L.

The nurse receives a laboratory report indicating a client's serum level is 1 mEq/L. The client's last dose of lithium was 8 hours ago. What does this result indicate? a. within therapeutic limits. b. below therapeutic limits. c. above therapeutic limits. d. invalid because of the time lapse since the last dose.

ANS: C,D People with mania are hyperactive, grandiose, and distractible. It is most important to ensure the client receives adequate nutrition. Structure will support a safe environment. Touching the client may precipitate aggressive behavior. Leading a community meeting would be appropriate when the client's behavior is less grandiose. Activities that require concentration will produce frustration.

The plan of care for a client in the manic state of bipolar disorder should include which interventions? (Select all that apply.) a. Touch the client to provide reassurance. b. Invite the client to lead a community meeting. c. Provide a structured environment for the client. d. Ensure that the client's nutritional needs are met. e. Design activities that require the client's concentration.

B. Risk for injury

What is the primary nursing concern related to a depressed client who has been taking citalopram for the past 3 weeks? A. Liver failure B. Risk for injury C. Blurred vision D. Urinary retention

ANS: A, D, E

Which findings are signs of a person who is mentally healthy? (Select all that apply.) a. Says, "I have some weaknesses, but I feel I'm important to my family and friends." b. Adheres strictly to religious beliefs of parents and family of origin. c. Spends all holidays alone watching old movies on television. d. Considers past experiences when deciding about the future. e. Experiences feelings of conflict related to changing jobs.

ANS: B All principles listed are valid, but the only rationale directly related to the intervention of assisting the client to talk about feelings and concerns is the one that states that concerns spoken aloud become less overwhelming and help problem solving begin.

A client is experiencing moderate anxiety. The nurse encourages the client to talk about feelings and concerns. What is the rationale for this intervention? a. Offering hope allays and defuses the client's anxiety. b. Concerns stated aloud become less overwhelming and help problem solving begin. c. Anxiety is reduced by focusing on and validating what is occurring in the environment. d. Encouraging clients to explore alternatives increases the sense of control and lessens anxiety.

ANS: C Sertraline is a selective serotonin reuptake inhibitor (SSRI). This antidepressant blocks the reuptake of serotonin, with few anticholinergic and sedating side effects. Clozapine is an antipsychotic. Chlordiazepoxide is an anxiolytic. Tacrine treats Alzheimer's disease.

A client is hospitalized for severe major depressive disorder. The nurse can expect to provide the client with teaching about what medication? a. chlordiazepoxide. b. clozapine. c. sertraline. d. tacrine.

ANS: D The duty to warn a person whose life has been threatened by a psychiatric client overrides the client's right to confidentiality. The right to confidentiality is not suspended at the discretion of the therapist or for legal investigations.

A family member of a client with delusions of persecution asks the nurse, "Are there any circumstances under which the treatment team is justified in violating a client's right to confidentiality?" What is the nurse's best response? a. Under no circumstances. b. At the discretion of the psychiatrist. c. When questions are asked by law enforcement. d. If the client threatens the life of another person.

ANS: A Sodium depletion and dehydration increase the chance for development of lithium toxicity. The other options offer inappropriate information.

A health teaching plan for a client taking lithium should include which instructions? a. maintain normal salt and fluids in the diet. b. drink twice the usual daily amount of fluid. c. double the lithium dose if diarrhea or vomiting occurs. d. avoid eating aged cheese, processed meats, and red wine.

B. Sedation Lorazepam is a benzodiazepine with anti-anxiety and sedative effects. Older adult clients are especially at risk for central nervous system depression, even with low doses of benzodiazepines. Clients who are 50 years or older can have more profound and prolonged sedation than younger clients.

A home health nurse is visiting an older adult client has Alzheimer's disease. His caregiver tells the nurse she has been administering prescribed lorazepam, 1 mg 3 times per day, to the client for restlessness and anxiety over the past few days. For which of the following adverse effects are the nurse assess the client? A. Low-grade fever B. Sedation C. Diuresis D. Tonic clonic seizures

ANS: B Sedation allows for safe withdrawal from alcohol. Benzodiazepines are the drugs of choice in most regions because of their high therapeutic safety index and anticonvulsant properties. The client's highest needs related to a need for calming.

A hospitalized client diagnosed with alcohol use disorder believes the window blinds are snakes trying to get in the room. The client is anxious, agitated, and diaphoretic. The nurse can anticipate the health care provider will prescribe what medication intervention? a. narcotic analgesic, such as hydromorphone. b. sedative, such as lorazepam or chlordiazepoxide. c. antipsychotic, such as olanzapine or thioridazine. d. monoamine oxidase inhibitor antidepressant, such as phenelzine.

ANS: b. sedative, such as lorazepam or chlordiazepoxide. Sedation allows for safe withdrawal from alcohol. Benzodiazepines are the drugs of choice in most regions because of their high therapeutic safety index and anticonvulsant properties. The client's highest needs related to a need for calming.

A hospitalized client diagnosed with alcohol use disorder believes the window blinds are snakes trying to get in the room. The client is anxious, agitated, and diaphoretic. The nurse can anticipate the health care provider will prescribe what medication intervention? a. narcotic analgesic, such as hydromorphone. b. sedative, such as lorazepam or chlordiazepoxide. c. antipsychotic, such as olanzapine or thioridazine. d. monoamine oxidase inhibitor antidepressant, such as phenelzine.

C. Tardiva dyskinesia Tar dive dyskinesia can be manifested by involuntary movements of many body parts. Early findings include withering movements of the tongue and smacking of the lips. The nurse report these manifestations to the provider immediately because the findings might not be reversible and can progress to affect all extremities. akathisia is a psycho motor restlessness that is usually manifested by fidgeting or pacing. Acute dystonia is manifested by severe muscle spasms are often occur in the head and neck. Pseudoparkinsonism is manifested by tremors of the extremities, loss of balance, and difficulty with gait.

A nurse is assessing a client who has schizophrenia and takes haloperidol three times daily. The client has developed involuntary withering movements of the tongue and constant lipsmacking. The nurse should identify that these manifestations indicate which of the following adverse effects of Haloperidol? A. Akathisia B. Acute dystonia C. Tardiva dyskinesia D. Pseudoparkinsonism

D. Paroxetine The nurse should expect the provider to prescribe paroxetine, and SSRI that is considered the first line treatment for PTSD. Bupropion is an aminoketone antidepressant that is Prescribed for smoking cessation, depression, and treatment of ADHD. It is not prescribed for the treatment of PTSD. Phenelzine is an MAOI antidepressant that can be prescribed for PTSD. However, SSRIs such as paroxetine are the first choice for PTSD. Mirtazapine is a tricyclic antidepressant that can be prescribed for PTSD. However SSRIs such as paroxetine are the first choice for PTSD

A nurse is assessing a client who is experiencing PTSD following a traumatic event. Which of the following medication to the nurse expect the provider to prescribe? A. Bupropion B. Phenelzine C. Mirtazapine D. Paroxetine

A. The client is experiencing mild acetaldehyde syndrome The nurse should recognize that these manifestations are an indication of acetaldehyde syndrome, which occurs when alcohol consumption is combined with the disulfiram use. The clients current manifestations represent the mild form of acetaldehyde syndrome that can occur by consuming as little as 7 mL of alcohol or 0.2 ounces.

A nurse is assessing a client who is receiving disulfiram for alcohol aversion therapy. The client is experiencing palpitations and reports nausea, headache, and extreme thirst. The nurse should identify that which of the following situation is occurring? A. The client is experiencing mild acetaldehyde syndrome B. The client is having delirium tremens C. The client is experiencing disulfiram toxicity D. The client is not having a therapeutic response to disulfiram

C. Xerostomia Buspirone can cause xerostomia or dry mouth. Other adverse effects include headache, nausea, and insomnia. Buspirone is more likely to cause myalgia than arthralgia. Buspirone is more likely to cause blurry vision than photophobia. Buspirone is more likely to cause tachycardia than bradycardia.

A nurse is assessing a client who is taking buspirone to treat generalized anxiety disorder. Which of the following findings should the nurse identify as an adverse effect of this medication? A. Arthralgia B. Photophobia C. Xerostomia (dry mouth) D. Bradycardia

D. Blurry vision Manifestations of lithium toxicity with levels between 2-2.5 include blurry vision, ataxia, chronic twitching, severe hypotension, and polyuria. Muscle weakness, find hand tremors, nausea, vomiting, diarrhea, and lethargic are early manifestations of lithium toxicity these manifestations are common with lifted lithium levels between 1.0 and 1.5 mEq/L. Manifestations of lithium toxicity between levels above 2.5 mEq/L include seizures and oliguria. For levels above 3.5 mg/dL: delirium, cardiovascular collapse, coma and death can occur.

A nurse is assessing a client who is taking lithium to treat bipolar disorder and has a lithium level of a 2.2 mEq/L. Which of the following findings should the nurse expect? A. Muscle weakness B. Oliguria C. Vomiting D. Blurry vision

A. Elevated blood pressure The greatest rest of this client is an elevated blood pressure, which increases the risk of hypertensive crisis that can result from taking an MAOI like phenelzine. The nurse should apply the safety and risk reduction priority setting framework when assessing this client, which assigns priority to the factor or situation posing the greatest safety risk to the client. When there are several risks to client safety, the rest posing the greatest threat is the highest priority. The nurse should use Maslow's hierarchy of needs, the ABC priority setting framework, and or nursing knowledge to identify which risk poses the greatest threat to the client. Weight gain muscle twitching and peripheral edema are all adverse effects of MAO eyes like phenelzine but elevated blood pressure takes priority.

A nurse is assessing a client who takes phenelzine for the treatment of depression. Which of the following findings is a priority for the nurse to report to provider? A. Elevated blood pressure B. Weight gain C. Muscle twitching D. 2+ peripheral edema

D. Monitor the client for escalating behavior Safety takes priority out of all the options

A nurse is caring for a client has bipolar disorder. Which of the following is the priority nursing action? A. Set consistent limits for expected client behavior B. Administer prescribed medication as scheduled C. Provide the client with step-by-step instructions during hygiene activities D. Monitor the client for escalating behavior

A. Ask the client to take a walk The clients increasing agitation demonstrates the potential for violent behavior. To maintain a safe environment, the nurse should remove the client from the situation and disperse the anger by walking and talking with her. The nurse should plan to administer a prescribed antianxiety medication such as lorazepam or an antipsychotic medication such as haloperidol if the clients behavior continues to escalate after less invasive interventions have been tried. Trazodone is a serotonin antagonist antidepressant medication, and it is not used for acute agitation. Reprimanding the client could be seen as confrontational the nurse should remain calm, speak in a quiet voice and demonstrated caring attitude towards this client. Any conversation between the nurse and the client about the clients agitated behavior should wait until the client is calm and able to discuss the incident.

A nurse is caring for a client who has bipolar disorder and is experiencing hypo mania. During a conversation with other clients, she becomes agitated and begin speaking in a loud, angry voice. Which of the following actions should the nurse take? A. Ask the client to take a walk B. Reprimand the client for her rude behavior C. Point out inappropriate behaviors to this client D. Administer trazodone to the client

B. "I am here to provide care and cannot accept this from you."

A nurse is caring for a client who has bipolar disorder. The client states, "I am very rich, and I feel I must give my money to you." Which of the following responses should the nurse make? A. "Why do you think you feel the need to give money away?" B. "I am here to provide care and cannot accept this from you." C. "I can request that your case manager discuss appropriate charity options with you." D. "You should know that giving away your money is inappropriate."

D. Poor motor coordination When using the urgent versus non-urgent approach to client care, the nurse should determine that the priority finding is poor motor coordination, which is an advanced manifestation of lithium toxicity. The nurse should hold the clients medication and notify the provider. Hand tremors are an expected finding for a client was recently been prescribed lithium, and the tremors may continue for weeks before subsiding. Weight gain and nausea are also unexpected findings for a client was recently been prescribed lithium.

A nurse is caring for a client who has recently been prescribed lithium carbonate. Which of the following assessment findings is a priority for this client? A. Find hand tremors B. Weight gain of 2.7 kg or 6 lbs C. Report of nausea D. Poor motor coordination

b. Discuss prior use of coping mechanisms with the client d. Demonstrate a calm manner while using simple and clear directions

A nurse is caring for a client who is experiencing moderate anxiety. Which of the following actions should the nurse take when trying to give necessary information to the client? SATA a. Reassure the client everything will be okay b. Discuss prior use of coping mechanisms with the client c. Ignore the client's anxiety so that she will not be embarrassed d. Demonstrate a calm manner while using simple and clear directions e. Gather information from the client using close-ended questions

C. Approach the client in a non-threatening manner The nurse should first approach the client commonly to create a non-threatening environment. The nurse should apply the least restrictive priority setting framework when caring for this client, which assigns priority to nursing interventions that are least restricted to the client, as long as these interventions do not jeopardize the client safety. Less restrictive interventions promote client safety without using restraints. The nurse should only use physical or chemical restraints when the safety of the client, staff members, or others are at risk.

A nurse is caring for a client who is schizophrenia. The nurse noticed that the client is pacing up and down the hall rapidly and muttering an angry manner. Which of the following actions should the nurse perform first? A. Apply mechanical restraints to the client B. Administer PRN haloperidol IM to the client C. Approach the client in a non-threatening manner D. Place the client in seclusion

B. Orthostatic hypotension D. Hypomania Observe for orthostatic hypotension and a headache when it comes to phenelzine. An elevated blood glucose is not an adverse effect of phenelzine. Priapism is an adverse effect of trazodone rather than phenelzine. Bruxism is a adverse effect of SSRIs rather than phenelzine.

A nurse is caring for a client who is taking phenelzine. For which of the following manifestations to the nurse monitor as an adverse affect of this medication? Select all that apply A. Elevated blood glucose level B. Orthostatic hypotension C. Priaprism D. Hypomania E. Bruxism

A. It is best to discontinue this medication slowly over 1 to 2 months. The nurse should respond by telling the client that withdrawing from the medication should be done slowly to reduce any manifestations of withdrawal. This can be achieved by reducing the dosage by 25% every one to two months. The nurse should respond by telling the client that the effects of fluoxetine take several months to Peak and that the therapy should continue for a year before discontinuation. The nurse should not indicate that weekend is likely with the withdrawal of fluoxetine. An adverse effect of taking this medication is waking. The nurse should not indicate that fluoxetine is only treatment for OCD. Other SSRI medication's are effective for this disorder. Some clients benefit from behavioral therapy and deep brain stimulation as well.

A nurse is caring for a client with OCD who has been taking fluoxetine for 3 months. The client states, "This medication isn't working. I want to stop taking it." Which of the following responses should the nurse make? A. It is best to discontinue this medication slowly over 1 to 2 months. B. If the medication hasn't helped you in 3 months, it's not going to. C. You will likely gain weight if you stop taking his medication. D. This medication is only treatment available for your condition.

b. A client who lives at home and keeps "forgetting" to come in for a scheduled monthly antipsychotic injection for schizophrenia

A nurse is caring for a group of clients. Which of the following clients should a nurse consider for referral to an assertive community treatment (ACT) group? a. A client in an acute care mental health facility who has fallen several times while running down the hallway b. A client who lives at home and keeps "forgetting" to come in for a scheduled monthly antipsychotic injection for schizophrenia c. A client in a day treatment program who reports anxiety during group therapy d. A client in a weekly grief support group who reports still missing a deceased partner who has been dead for 3 months

d. Remove the patch each day after 9 hr The transdermal patch is applied once daily in the morning and is removed after 9 hours

A nurse is caring for a school age child who has conduct disorder and a new prescription for methylphenidate transdermal patches. Which of the following information should the nurse provide about the medication? a. Apply the patch once daily at bedtime b. Place the patch carefully in a trash can after removal c. Apply the transdermal patch to the anterior waist area d. Remove the patch each day after 9 hr

C. Electrocardiogram Amitriptyline can cause tachycardia and ECG changes. An older adult client is at risk for cardiovascular effects while using amitriptyline, therefore, an ECG should be performed prior to the start of therapy to obtain a baseline of the clients cardiovascular status. Amitriptyline can cause blurred vision however it does not affect hearing. Enemy tripped when does not cause hyperglycemia or alter glucose tolerance. It also does not affect pulmonary function.

A nurse is caring for an older adult client who has a new prescription for amitriptyline to treat depression. Which of the following diagnostic test should a nurse plan to perform prior to starting the client on this medication? A. Hearing examination B. Glucose tolerance test C. Electrocardiogram D. Pulmonary function test

C. Smoked fish MAOI teaching includes that clients should avoid tyramine

A nurse is providing teaching to a client with a new prescription for Selegine. The nurse will instruct the client to avoid which food? A. Canned pineapple B. Frozen peas C. Smoked fish D. Cottage cheese

ANS: B, C People with mania are hyperactive and often do not take time to eat and drink properly. Their high levels of activity consume calories, so deficits in nutrition may occur. The mood evidences euphoria and is labile. Sleep is reduced. Their socialization is impaired but not isolated. Confusion may be acute but not chronic.

A nurse prepares the plan of care for a client experiencing an acute manic episode. Which nursing diagnoses are most likely? (Select all that apply.) a. Imbalanced nutrition: more than body requirements b. Impaired mood regulation c. Sleep deprivation d. Chronic confusion e. Social isolation

ANS: B Splitting involves loving a person, then hating the person because the client is unable to recognize that an individual can have both positive and negative qualities. Denial is unconsciously motivated refusal to believe something. Reaction formation involves unconsciously doing the opposite of a forbidden impulse. The scenario does not indicate defensiveness.

A nurse set limits while interacting with a client demonstrating behaviors associated with borderline personality disorder. The client tells the nurse, "You used to care about me. I thought you were wonderful. Now I can see I was wrong. You're evil." This outburst can be documented using what term? a. denial. b. splitting. c. defensive. d. reaction formation.

A. The patient who recurses to eat or bathe

A nurse uses Maslow's hierarchy of needs to plan care for a patient diagnosed with mental illness. Which problem will receive priority? A. The patient who recurses to eat or bathe B. The patient who reports feelings of alienation from family C. The patient who is reluctant to participate in unit social activities D. The patient who is unaware of medication action and side effects

ANS: A Teaching about symptoms of anxiety, their relation to precipitating stressors, and, in this case, the positive effects of anxiety will serve to reassure the client. Advising the client to discuss the experience with a health care provider implies that the client has a serious problem. Listening without comment will do no harm but deprives the client of health teaching. Antioxidant vitamin supplements are not useful in this scenario.

A student says, "Before taking a test, I feel very alert and a little restless." Which nursing intervention is most appropriate to assist the student? a. Explain that the symptoms result from mild anxiety and discuss the helpful aspects. b. Advise the student to discuss this experience with a health care provider. c. Encourage the student to begin antioxidant vitamin supplements. d. Listen attentively, using silence in a therapeutic way.

ANS: D As an individual accepts loss, the person renews interest in people and activities. The person is seeking to move into new relationships. The patient's comment demonstrates an attempt to regain control. Bargaining is evidenced by people reviewing what could have been done differently. While the person may also experience occasional anger or sadness, the comment speaks directly to acceptance.

A widower tells friends, "I am taking my neighbor out for dinner. It's time for me to be more sociable again." Considering the stages of grief described by Kübler-Ross, which stage is evident? a. Anger b. Denial c. Depression d. Acceptance

ANS: C Strong positive or negative reactions toward a patient or over-identification with the patient indicate possible countertransference. Nurses must carefully monitor their own feelings and reactions to detect countertransference and then seek supervision. Realistic and appropriate reactions from a patient toward a nurse are desirable. One incorrect response suggests transference. A trusting relationship with the patient is desirable. See relationship to audience response question.

After several therapeutic encounters with a patient who recently attempted suicide, which occurrence should cause the nurse to consider the possibility of countertransference? a. The patient's reactions toward the nurse seem realistic and appropriate. b. The patient states, "Talking to you feels like talking to my parents." c. The nurse feels unusually happy when the patient's mood begins to lift. d. The nurse develops a trusting relationship with the patient.

ANS: B Redirecting the expression of feelings into nondestructive, age-appropriate behaviors such as a physical activity helps the child learn how to modulate the expression of feelings and exert self- control. This is the least restrictive alternative and should be tried before resorting to measures that are more restrictive. A shouting child will not likely engage in a discussion about feelings. A behavioral contract could be considered later, but first the situation must be defused.

An 11-year-old diagnosed with oppositional defiant disorder (ODD) becomes angry over the rules at a residential treatment program and begins shouting at the nurse. What is the nurse's initial action to defuse the situation? a. Say to the child, "Tell me how you're feeling right now." b. Take the child swimming at the facility's pool. c. Establish a behavioral contract with the child. d. Administer an anxiolytic medication.

ANS: A Diphenhydramine, trihexyphenidyl, benztropine, and other anticholinergic medications may be used to treat dystonias. Swallowing will be difficult or impossible; therefore, oral medication is not an option. Medication should be administered immediately, so the intramuscular route is best. In this case, the best option given is diphenhydramine.

An acutely violent client diagnosed with schizophrenia received several doses of haloperidol. Two hours later the nurse notices the client's head rotated to one side in a stiffly fixed position, the lower jaw thrust forward, and drooling. Which intervention by the nurse is indicated? a. Administer diphenhydramine 50 mg IM from the prn medication administration record. b. Reassure the client that the symptoms will subside. Practice relaxation exercises with the client. c. Give trihexyphenidyl 5 mg orally at the next regularly scheduled medication administration time. d. Administer atropine sulfate 2 mg subcut from the prn medication administration record.

ANS: B Breach of nurse-client confidentiality does not pose a legal dilemma for nurses in these circumstances because a team approach to delivery of psychiatric care presumes communication of client information to other staff members to develop treatment plans and outcome criteria. The client should also know that the team has a duty to warn the father of the risk for harm.

An adolescent hospitalized after a violent physical outburst tells the nurse, "I'm going to kill my father, but you can't tell anyone." What is the nurse's best response? a. "You are right. Federal law requires me to keep clinical information private." b. "I am obligated to share that information with the treatment team." c. "Those kinds of thoughts will make your hospitalization longer." d. "You should share this thought with your psychiatrist."

ANS: C The parents are seeking a quick solution. Medications are generally not indicated for ODD. Comorbid conditions that increase defiant symptoms, such as ADHD, should be managed with medication, but no comorbid problem is identified in the question. The nurse should give information on helpful strategies to manage the adolescent's behavior.

An adolescent was recently diagnosed with oppositional defiant disorder (ODD). The parents say to the nurse, "Isn't there some medication that will help with this problem?" What is the nurse's best response? a. "There are no medications to treat this problem. This diagnosis is behavioral in nature." b. "It's a common misconception that there is a medication available to treat every health problem." c. "Medication is usually not prescribed for this problem. Let's discuss some behavioral strategies you can use." d. "There are many medications that will help your child manage aggression and destructiveness. The health care provider will discuss them with you."

ANS: D The patient's comment demonstrates an attempt to regain control. Bargaining is evidenced by people reviewing what could have been done differently. While the person may also be experiencing anger and depression, the comment speaks directly to bargaining. The person shows acceptance of the disease.

An adult says to the nurse, "The cancer in my neck spread in only 2 months. I've been cursed my whole life. Maybe if I had been more generous with others ..." Considering the stages of grief described by Kübler-Ross, which stage is evident? a. Anger b. Denial c. Depression d. Bargaining

ANS: C In the middle stage, deterioration is evident. Memory loss may include the inability to remember addresses or the date. Activities such as driving may become hazardous, and frustration by the increasing difficulty of performing ordinary tasks may be experienced. The individual has difficulty with clothing selection. Mild cognitive decline (early-stage) Alzheimer's can be diagnosed in some, but not all, individuals. Symptoms include misplacing items and misuse of words. In the late stage there is severe cognitive decline along with agraphia, hyperorality, blunting of emotions, visual agnosia, and hypermetamorphosis. Sundowning is not a stage of Alzheimer's disease.

An older adult drove to a nearby store but was unable to remember how to get home or state an address. When police intervened, they found that this adult was wearing a heavy coat and hat, even though it was July. Which stage of Alzheimer's disease is evident? a. Sundowning b. Early c. Middle d. Late

ANS: C Agnosia refers to the loss of sensory ability to recognize objects. Aphasia refers to the loss of language ability. Apraxia refers to the loss of purposeful movement. Anhedonia refers to a loss of joy in life.

An older adult was stopped by police for driving through a red light. When asked for a driver's license, the adult hands the police officer a pair of sunglasses. What sign of dementia is evident? a. Aphasia b. Apraxia c. Agnosia d. Anhedonia

ANS: B. Valproate The medications listed in the stem are mood stabilizers, anticonvulsant types. Valproate (Depakote) is also a member of this group. The distracters are drugs for treatment of Alzheimer's disease and anxiety.

Consider these medications: carbamazepine, lamotrigine, gabapentin. Which medication below also belongs to this group? a. Galantamine b. Valproate c. Buspirone d. Tacrine

C. Neuroleptic malignant syndrome Look all the symptoms, acute dystonia doesn't have elevated temperature even though it does have muscle rigidity

One week after beginning therapy with olanzapine, the client demonstrates muscle rigidity, a temperature of 103 Fahrenheit, stupor, and incontinence. The nurse should notify the physician because the symptoms are indicative of what? A. Acute dystonic reaction B. Extrapyramidal side effects C. Neuroleptic malignant syndrome D. Tardive dyskinesia

ANS: A. report to provider These laboratory values indicate the possibility of agranulocytosis, a serious side effect of clozapine therapy. These results must be immediately reported to the health care provider, and the drug should be withheld. The health care provider may repeat the test, but in the meantime, the drug should be withheld.

The laboratory report for a client taking clozapine shows a white blood cell count of 3000 mm3. What is the nurse's best action? a. Report the results to the health care provider immediately. b. Administer the next dose as prescribed. c. Give aspirin and force fluids. d. Repeat the laboratory test.

D. "Tell me about your plan."

The nurse is caring for a client diagnosed with a major depressive disorder. The client states, "As soon as I get out of here I'm going to kill myself." What is the nurses first response? A. "Tell me more about how you are feeling." B. "I am sure you will your mind by then." C. "You have so much to live for." D. "Tell me about your plan."

ANS: A Lithium is a salt and known to alter fluid and electrolyte balance, producing polyuria, edema, and other symptoms of imbalance. Clients receiving clozapine should be monitored for agranulocytosis. Clients receiving fluoxetine should be monitored for acetylcholine block. Clients receiving venlafaxine should be monitored for heightened feelings of anxiety.

The nurse should be most alert for problems associated with fluid and electrolyte imbalance when a client is prescribed which medication? a. lithium. b. clozapine. c. fluoxetine. d. venlafaxine.

ANS: C Clients taking phenelzine, a monoamine oxidase (MAO) inhibitor, must be on a low tyramine diet to prevent hypertensive crisis. There are no specific dietary precautions associated with the distracters.

The nurse will order a special diet for the client who is prescribed which medication? a. carbamazepine. b. haloperidol. c. phenelzine. d. trazodone.

ANS: C Drinking alcohol or taking other anxiolytics along with the prescribed benzodiazepine should be avoided because depressant effects of both drugs will be potentiated. Tyramine-free diets are necessary only with monoamine oxidase inhibitors (MAOIs). Drowsiness is an expected effect and needs to be reported only if it is excessive. Clients should be taught not to deviate from the prescribed dose and schedule for administration.

When alprazolam is prescribed for a client who experiences acute anxiety, health teaching should include which instruction? a. report drowsiness. b. eat a tyramine-free diet. c. avoid alcoholic beverages. d. adjust dose and frequency based on anxiety level.

A. Exploring alternative solutions with the patient, who then makes a choice.

Which action by psychiatric nurse best applies the ethical principle of autonomy? A. Exploring alternative solutions with the patient, who then makes a choice. B. Suggest that two patients who were fighting be restricted to the unit. C. Intervening when a self-mutilating patient attempts to harm self. D. Staying with a patient demonstrating a high level of anxiety

B. "He deserved it for being a sissy." They justify the behavior

Which statement made by a 9 yr old child after hitting another child is a typical comment associated with childhood conduct disorder? A. "I'm sorry, I wont hit him again." B. "He deserved it for being a sissy." C. "I didnt think I hit him very hard." D. "I don't have to do what you say."

ANS: D CDs are manifested by a persistent pattern of behavior in which the rights of others and age- appropriate societal norms are violated. Intermittent explosive disorder is a pattern of behavioral outbursts characterized by an inability to control aggressive impulses in adults 18 years and older. Criteria for ADHD and PTSD are not met in the scenario

A 15-year-old ran away from home six times and was arrested for shoplifting. The parents told the Court, "We can't manage our teenager." The adolescent is physically abusive to the mother and defiant with the father. Which diagnosis is supported by this adolescent's behavior? a. Attention deficit hyperactivity disorder (ADHD) b. Posttraumatic stress disorder (PTSD) c. Intermittent explosive disorder d. Conduct disorder (CD)

ANS: A The adolescent and the parents must agree on a behavioral contract that clearly outlines rules, expected behaviors, and consequences for misbehavior. It must also include rewards for following the rules. The adolescent will continue to experience anger and frustration. The adolescent and parents must continue with family therapy to work on boundary and communication issues. It is not necessary to separate the adolescent from the family to work on these issues. Separation is detrimental to the healing process. While it is helpful for the adolescent to identify peers who are a positive influence, it is more important for behavior to be managed for an adolescent diagnosed with a CD.

A 16-year-old diagnosed with a conduct disorder (CD) has been in a residential program for 3 months. Which outcome should occur before discharge? a. The adolescent and parents create and agree to a behavioral contract with rules, rewards, and consequences. b. The adolescent identifies friends in the home community who are a positive influence. c. Temporary placement is arranged with a foster family until the parents complete a parenting skills class. d. The adolescent experiences no anger and frustration for 1 week.

ANS: C The goal is improvement in the child's hyperactivity, aggression, and play. The remaining options are more relevant for a child with intellectual development disorder or an anxiety disorder.

A child diagnosed with attention deficit hyperactivity disorder (ADHD) shows hyperactivity, aggression, and impaired play. The health care provider prescribed amphetamine salts. The nurse should monitor for which desired behavior? a. Increased expressiveness in communication with others b. Abilities to identify anxiety and implement self-control strategies c. Improved abilities to participate in cooperative play with other children d. Tolerates social interactions for short periods without disruption or frustration

ANS: A CNS stimulants, such as methylphenidate and pemoline, increase blood flow to the brain and have proved helpful in reducing hyperactivity in children and adolescents with ADHD. The other medication categories listed would not be appropriate.

A child diagnosed with attention deficit hyperactivity disorder (ADHD) will begin medication therapy. The nurse should prepare a plan to teach the family about which classification of medications? a. CNS stimulants b. Tricyclic antidepressants c. Antipsychotics d. Anxiolytics

ANS: A Minimizing one's drinking is a form of denial of alcoholism. The Client is more than a social drinker. Projection involves blaming another for one's faults or problems. Rationalization involves making excuses. Introjection involves incorporating a quality of another person or group into one's own personality.

A client admitted to an alcohol rehabilitation program tells the nurse, "I'm actually just a social drinker. I usually have a drink at lunch, two in the afternoon, wine with dinner, and a few drinks during the evening." The client is using which defense mechanism? a. Denial b. Projection c. Introjection d. Rationalization

ANS: C The client has symptoms associated with abrupt withdrawal of the antidepressant. Taking a dose of the drug will ameliorate the symptoms. Seeing the health care provider will allow the client to discuss the advisability of going off the medication and to be given a gradual withdrawal schedule if discontinuation is the decision. This situation is not a medical emergency, although it calls for medical advice. Resuming taking the antidepressant for 2 more weeks and then discontinuing again would produce the same symptoms the client is experiencing.

A client being treated for depression has taken sertraline daily for a year. The client calls the clinic nurse and says, "I stopped taking my antidepressant 2 days ago. Now I am having nausea, nervous feelings, and I can't sleep." The nurse will advise the client to: a. "Go to the nearest emergency department immediately." b. "Do not to be alarmed. Take two aspirin and drink plenty of fluids." c. "Take a dose of your antidepressant now and come to the clinic to see the health care provider." d. "Resume taking your antidepressants for 2 more weeks and then discontinue them again."

ANS: A, D, E The client is taking the maximum dose of this SSRI and has ingested an additional unknown amount of the drug. Serotonin syndrome must be considered. Symptoms include abdominal pain, diarrhea, tachycardia, elevated blood pressure, hyperpyrexia, increased motor activity, and muscle spasms. Serotonin syndrome may progress to a full medical emergency if not treated early. The client may have urinary retention, but frequency would not be expected.

A client being treated with paroxetine 50 mg po daily reports to the clinic nurse, "I took a few extra tablets earlier today and now I feel bad." Which assessments are most critical? (Select all that apply.) a. Vital signs b. Urinary frequency c. Psychomotor retardation d. Presence of abdominal pain and diarrhea e. Hyperactivity or feelings of restlessness

ANS: B Manic symptoms are controlled by lithium only after a therapeutic serum level is attained. Because this takes several days to accomplish, a drug with rapid onset is necessary to reduce the hyperactivity initially. Antipsychotic drugs neither enhance lithium's antimanic activity nor minimize the side effects. Lithium will be used for long-term control.

A client demonstrating characteristics of acute mania relapsed after discontinuing lithium. New orders are written to resume lithium twice daily and begin olanzapine. What is the rationale for the addition of olanzapine to the medication regimen? a. To minimize the side effects of lithium. b. To bring hyperactivity under rapid control. c. To enhance the antimanic actions of lithium. d. To be used for long-term control of hyperactivity.

ANS: D Agnosia is the inability to recognize familiar objects, parts of one's body, or one's own reflection in a mirror. Hyperorality refers to placing objects in the mouth. Aphasia refers to the loss of language ability. Apraxia refers to the loss of purposeful movements, such as being unable to dress.

A client diagnosed with Alzheimer's disease calls the fire department saying, "My smoke detectors are going off." Firefighters investigate and discover that the client misinterpreted the telephone ringing. Which problem is this client experiencing? a. Hyperorality b. Aphasia c. Apraxia d. Agnosia

ANS: D A client who repeatedly disrobes despite verbal limit setting needs more structure. One-on-one supervision may provide the necessary structure. Directing the client to wear clothes at all times has not proven successful, considering the behavior has continued. Asking if the client is bothered by clothing serves no purpose. Telling the client that others are embarrassed will not make a difference to the client whose grasp of social behaviors is impaired by the illness.

A client diagnosed with acute mania has disrobed in the hall three times in 2 hours. What intervention should the nurse implement? a. direct the client to wear clothes at all times. b. ask if the client finds clothes bothersome. c. tell the client that others feel embarrassed. d. arrange for one-on-one supervision.

ANS: C When the client is unable to control his or her behavior and violates or threatens to violate the rights of others, limits must be set in an effort to de-escalate the situation. Limits should be set in simple, concrete terms. The incorrect responses do not offer appropriate assistance to the client, threaten the client with seclusion as punishment, and ask a rhetorical question.

A client diagnosed with bipolar disorder becomes hyperactive after discontinuing lithium. The client threatens to hit another client. Which comment by the nurse is appropriate? a. "Stop that! No one did anything to provoke an attack by you." b. "If you do that one more time, you will be secluded immediately." c. "Do not hit anyone. If you are unable to control yourself, we will help you." d. "You know we will not let you hit anyone. Why do you continue this behavior?"

ANS: D Some clients diagnosed with bipolar disorder, especially those who have only short periods between episodes, have a favorable response to the anticonvulsants carbamazepine and valproate. Carbamazepine seems to work better in clients with rapid cycling and in severely paranoid, angry manic clients. Phenytoin is also an anticonvulsant but not used for mood stabilization. Risperidone is not an anticonvulsant.

A client diagnosed with bipolar disorder has rapidly changing mood cycles. The health care provider prescribes an anticonvulsant medication. To prepare teaching materials, which drug should the nurse anticipate will be prescribed? a. phenytoin b. clonidine c. risperidone d. carbamazepine

ANS: B Clients diagnosed with bipolar disorder may be maintained on lithium indefinitely to prevent recurrences. Helping the client understand this need will promote medication adherence

A client diagnosed with bipolar disorder is in the maintenance phase of treatment. The client asks, "Do I have to keep taking this lithium even though my mood is stable now?" What is the nurse's most appropriate response? a. "You will be able to stop the medication in about 1 month." b. "Taking the medication every day helps reduce the risk of a relapse." c. "Most clients take medication for approximately 6 months after discharge." d. "It's unusual that the health care provider hasn't already stopped your medication."

ANS: B The symptoms described suggest lithium toxicity. The client should have a lithium level drawn and may require further treatment. Because neurological symptoms are present, the client should not drive and should be accompanied by another person. The incorrect options will not ameliorate the client's symptoms.

A client diagnosed with bipolar disorder is prescribed lithium. The client telephones the nurse to say, "I've had severe diarrhea for 4 days. I feel very weak and unsteady when I walk. My usual hand tremor has gotten worse. What should I do?" What advise will they give to the client? a. restrict food and fluids for 24 hours and stay in bed. b. have someone bring the client to the clinic immediately. c. drink a large glass of water with 1 teaspoon of salt added. d. take one dose of an over-the-counter antidiarrheal medication now.

ANS: A The scenario describes self-mutilation. Self-mutilation is a nursing diagnosis relating to client safety needs and is therefore of high priority. Impaired skin integrity and powerlessness may be appropriate foci for care but are not the priority related to this therapy. Risk for injury implies accidental injury, which is not the case for the client with borderline personality disorder.

A client diagnosed with borderline personality disorder was hospitalized several times after multiple episodes of head banging and carving on both wrists. The client remains impulsive. Which nursing diagnosis is the initial focus of this client's care? a. Self-mutilation b. Impaired skin integrity c. Risk for injury d. Powerlessness

ANS: C Escitalopram is an SSRI antidepressant. One to three weeks of treatment is usually necessary before symptom relief occurs. This information is important to share with clients.

A client diagnosed with major depressive disorder began taking escitalopram 5 days ago. The client now says, "This medicine isn't working." What is the nurse's best intervention? a. discuss with the health care provider the need to increase the dose. b. reassure the client that the medication will be effective soon. c. explain the time lag before antidepressants relieve symptoms. d. critically assess the client for symptoms of improvement.

ANS: D. Urinary retention All the side effects mentioned are the result of the anticholinergic effects of the drug. Only urinary retention and severe constipation warrant immediate medical attention. Dry mouth, blurred vision, and nasal congestion may be less troublesome as therapy continues.

A client diagnosed with major depressive disorder is receiving imipramine 200 mg at bedtime. Which assessment finding would prompt the nurse to collaborate with the health care provider regarding potentially hazardous side effects of this drug? a. Dry mouth b. Blurred vision c. Nasal congestion d. Urinary retention

ANS: A, B, E Providing clothing with elastic and hook-and-loop closures facilitates client independence. Labeling clothing with the client's name and the name of the item maintains client identity and dignity (provides information if the client has agnosia). When a client resists, it is appropriate to use distraction and try again after a short interval because client's moods are often labile. The client may be willing to cooperate given a later opportunity. Providing the necessary items for grooming and directing the client to proceed independently are inappropriate. Be prepared to coach by giving step-by-step directions for each task as it occurs. Administering anxiolytic medication before bathing and dressing is inappropriate. This measure would result in unnecessary overmedication.

A client diagnosed with moderate stage Alzheimer's disease has a self-care deficit of dressing and grooming. Designate appropriate interventions to include in the client's plan of care. (Select all that apply.) a. Provide clothing with elastic and hook-and-loop closures. b. Label clothing with the client's name and name of the item. c. Administer antianxiety medication before bathing and dressing. d. Provide necessary items and direct the client to proceed independently. e. If the client resists dressing, use distraction and try again after a short interval.

ANS: B It is the health care professional's duty to warn or notify an intended victim after a threat of harm has been made. Informing a potential victim of a threat is a legal responsibility of the health care professional. None of the other options are true statements.

A client diagnosed with schizophrenia believes a local minister has stirred evil spirits and threatens to bomb a local church. The psychiatrist notifies the minister based on what rationale? a. The psychiatrist may release information at their discretion. b. The psychiatrist demonstrated the duty to warn and protect. c. The psychiatrist has no obligation concerning the client's confidentiality. d. The psychiatrist is immune from charges of malpractice.

ANS: C Clients do not stay in a hospital until every symptom disappears. The nurse must assume responsibility to advocate for the client's right to the least restrictive setting as soon as the symptoms are under control and for the right of citizens to control health care costs. The health care provider will use the same rationale. Shifting blame will not change the discharge. Security is unnecessary. The nurse can handle this matter.

A client diagnosed with schizophrenia had an exacerbation related to medication non-adherence and was hospitalized for 5 days. The client's thoughts are now more organized, and discharge is planned. The client's family says, "It's too soon for discharge. We will just go through all this again." What action should the nurse take? a. ask the case manager to arrange a transfer to a long-term care facility. b. notify hospital security to handle the disturbance and escort the family off the unit. c. explain that the client will continue to improve if the medication is taken regularly. d. contact the health care provider to meet with the family and explain the discharge

ANS: A The nurse should use the client's support system to meet client needs whenever possible. Delivery of medication by the nurse should be unnecessary for the nurse to do if client or a significant other can be responsible. The client may not need more intensive follow-up as long as medication is taken as prescribed.

A client diagnosed with schizophrenia has been stable for 2 months. Today the client's spouse calls the nurse to report the client has not taken prescribed medication and is having disorganized thinking. The client forgot to refill the prescription. The nurse arranges a refill. What is the best outcome to add to the plan of care? a. The client's spouse will mark dates for prescription refills on the family calendar. b. The nurse will obtain prescription refills every 90 days and deliver to the client. c. The client will call the nurse weekly to discuss medication-related issues. d. The client will report to the clinic for medication follow-up every week.

ANS: D Signs of potential relapse include feeling tense, difficulty concentrating, trouble sleeping, increased withdrawal, and increased bizarre or magical thinking. Medication nonadherence may not be implicated. Relapse can occur even when the client is taking medication regularly. Psychoeducation is more effective when the client's symptoms are stable. Chronic deterioration is not the best explanation.

A client diagnosed with schizophrenia has been stable for a year; however, the family now reports the client is tense, sleeps 3 to 4 hours per night, and has difficulty concentrating. The client says, "My computer is sending out infected radiation beams." The nurse can correctly assess this information as an indication of what? a. the need for psychoeducation. b. medication nonadherence. c. chronic deterioration. d. relapse.

ANS: B Olanzapine is a second-generation atypical antipsychotic that targets both positive and negative symptoms of schizophrenia. Haloperidol and chlorpromazine are conventional antipsychotics that target only positive symptoms. Diphenhydramine is an antihistamine.

A client diagnosed with schizophrenia has taken a conventional antipsychotic medication for a year. Hallucinations are less intrusive, but the client continues to have apathy, poverty of thought, and social isolation. The nurse would expect a change to which medication? a. Haloperidol b. Olanzapine c. Chlorpromazine d. Diphenhydramine

ANS: A Acute dystonic reactions involve painful contractions of the tongue, face, neck, and back. Opisthotonos and oculogyric crisis may be observed. Dystonic reactions are considered emergencies requiring immediate intervention. Tardive dyskinesia involves involuntary spasmodic muscular contractions that involve the tongue, fingers, toes, neck, trunk, or pelvis. It appears after prolonged treatment. Waxy flexibility is a symptom seen in catatonic schizophrenia. Internal and external restlessness, pacing, and fidgeting are characteristics of akathisia.

A client diagnosed with schizophrenia is very disturbed and violent. After several doses of haloperidol, the client is calm. Two hours later the nurse sees the client's head rotated to one side in a stiff position, the lower jaw thrust forward, and drooling. Which problem is most likely? a. An acute dystonic reaction b. Tardive dyskinesia c. Waxy flexibility d. Akathisia

ANS: D Looseness of association refers to jumbled thoughts incoherently expressed to the listener. Neologisms are newly coined words. Ideas of reference are a type of delusion. Thought broadcasting is the belief that others can hear one's thoughts

A client diagnosed with schizophrenia says, "It's beat. Time to eat. No room for the cat." What type of verbalization is evident? a. Neologism b. Idea of reference c. Thought broadcasting d. Associative looseness .

ANS: D When a client's speech is loosely associated, confused, and disorganized, pretending to understand is useless. The nurse should tell the client that he or she is having difficulty understanding what the client is saying. If a theme is discernible, ask the client to talk about the theme. The incorrect options tend to place blame for the poor communication with the client. The correct response places the difficulty with the nurse rather than being accusatory.

A client diagnosed with schizophrenia tells the nurse, "I eat skiller. Tend to end. Easter. It blows away. Get it?" What is the nurse's most therapeutic response? a. "Nothing you are saying is clear." b. "Your thoughts are very disconnected." c. "Try to organize your thoughts and then tell me again." d. "I am having difficulty understanding what you are saying."

ANS: B It is important not to challenge the client's beliefs, even if they are unrealistic. Challenging undermines the client's trust in the nurse. The nurse should try to understand the underlying feelings or thoughts the client's message conveys. The correct response uses the therapeutic technique of reflection. The other comments are nontherapeutic. Asking to talk about something other than the concern at hand is changing the subject. Saying that the CIA is prohibited from operating in health care facilities gives false reassurance. Stating that the client has lost touch with reality is truthful but uncompassionate.

A client diagnosed with schizophrenia tells the nurse, "The Central Intelligence Agency is monitoring us through the fluorescent lights in this room. The CIA is everywhere, so be careful what you say." Which response by the nurse is most therapeutic? a. "Let's talk about something other than the CIA." b. "It sounds like you're concerned about your privacy." c. "The CIA is prohibited from operating in health care facilities." d. "You have lost touch with reality, which is a symptom of your illness."

ANS: B Lorazepam is a benzodiazepine used to treat anxiety. It may be given as a prn medication. Buspirone is long acting and is not useful as a prn drug. Amitriptyline and desipramine are tricyclic antidepressants and considered second- or third-line agents.

A client experiences a sudden episode of severe anxiety. Of these medications in the client's medical record, which is most appropriate to give as a prn anxiolytic? a. buspirone b. lorazepam c. amitriptyline d. desipramine

ANS: B The client's behavior demonstrates a clear risk of dangerousness to others. Safety is of primary importance. Once other clients are out of the room, a plan for managing this client can be implemented. Threatening the client or assembling a show of force is likely to exacerbate the tension.

A client experiencing acute mania is dancing atop a pool table in the recreation room. The client waves a cue in one hand and says, "I'll throw the pool balls if anyone comes near me." To best assure safety, what is the nurse's first intervention? a. tell the client, "You need to be secluded." b. clear the room of all other clients. c. help the client down from the table. d. assemble a show of force.

ANS: C Clients must be protected from the embarrassing consequences of their poor judgment whenever possible. Protecting the client from public exposure by matter-of-factly covering the client and removing him or her from the area with a sufficient number of staff to avoid argument and provide control is an effective approach.

A client experiencing acute mania undresses in the group room and dances. How should the nurse intervene initially? a. quietly asking the client, "Why don't you put your clothes on?" b. firmly telling the client, "Stop dancing and put on your clothing." c. putting a blanket around the client and walking with the client to a quiet room. d. letting the client stay in the group room and moving the other clients to a different area.

ANS: C Safety needs of the client and other clients are a priority. Comments to the client should be simple, neutral, and give direction to help the client regain control. Running after the client will increase the client's anxiety. More than one staff member may be needed to provide physical limits but using seclusion or physically restraining the client prematurely is unjustified. Asking the client to give an example would be futile; a client in panic processes information poorly.

A client experiencing panic suddenly began running and shouting, "I'm going to explode!" What is the nurse's best action? a. Ask, "I'm not sure what you mean. Give me an example." b. Capture the client in a basket-hold to increase feelings of control. c. Tell the client, "Stop running and take a deep breath. I will help you." d. Assemble several staff members and say, "We will take you to seclusion to help you regain control."

ANS: A Safety is of highest priority because the client experiencing panic is at high risk for self-injury related to increased non-goal-directed motor activity, distorted perceptions, and disordered thoughts. Offering an outlet for the client's energy can occur when the current panic level subsides. Respecting the client's personal space is a lower priority than safety. Clarification of feelings cannot take place until the level of anxiety is lowered.

A client fearfully runs from chair to chair crying, "They're coming! They're coming!" The client does not follow the staff's directions or respond to verbal interventions. What is the initial nursing intervention of highest priority? a. providing for the client's safety. b. encouraging clarification of feelings. c. respecting the client's personal space. d. offering an outlet for the client's energy.

ANS: A Clients with cognitive impairment should perform all tasks of which they are capable. When simple directions are given in a systematic fashion, the client is better able to process information and perform simple tasks. Stimulating intellectual functioning by discussing new topics is likely to prove frustrating for the client. Clients with cognitive deficits may enjoy the attention of someone reading to them, but this activity does not promote their function in the environment.

A client has progressive memory deficits associated with dementia. Which nursing intervention would best help the individual function in the environment? a. Assist the client to perform simple tasks by giving step-by-step directions. b. Reduce frustration by performing activities of daily living for the client. c. Stimulate intellectual function by discussing new topics with the client. d. Read one story from the newspaper to the client every day.

ANS: B Systematic desensitization is a form of behavior modification therapy that involves the development of behavior tasks customized to the client's specific fears. These tasks are presented to the client while using learned relaxation techniques. The client is incrementally exposed to the fear.

A client is fearful of riding on elevators. The therapist first rides an escalator with the client. The therapist and client then stand in an elevator with the door open for 5 minutes and later with the elevator door closed for 5 minutes. Which technique has the therapist used? a. Classic psychoanalytic therapy b. Systematic desensitization c. Rational emotive therapy d. Biofeedback

ANS: D The physical symptoms are consistent with central nervous system (CNS) stimulation. Suspicion and paranoid ideation are also present. Amphetamine use is likely. PCP use would probably result in bizarre, violent behavior. Barbiturates and heroin would result in symptoms of CNS depression.

A client is thin, tense, jittery, and has dilated pupils. The client says, "My heart is pounding in my chest. I need help." The client allows vital signs to be taken but then becomes suspicious and says, "You could be trying to kill me." The client refuses further examination. Abuse of which substance is most likely? a. phencyclidine (PCP) b. Heroin c. Barbiturates d. Amphetamines

ANS: D The client with heart failure will likely need diuretic drugs, which will complicate the maintenance of the fluid balance necessary to avoid lithium toxicity. None of the other options would present such a challenge.

A client newly diagnosed with bipolar disorder is prescribed lithium. Which information from the client's medical history indicates that monitoring of serum concentrations of the drug will be challenging and critical? a. Arthritis b. Epilepsy c. Psoriasis d. Heart failure

ANS: B Because obsessive-compulsive clients become overly involved in the rituals, promotion of involvement with other people and activities is necessary to improve coping. Daily activities prevent constant focus on anxiety and symptoms. The other interventions focus on the compulsive symptom.

A client performs ritualistic hand washing. Which action should the nurse implement to help the client develop more effective coping? a. Allow the client to set a hand-washing schedule. b. Encourage the client to participate in social activities. c. Encourage the client to discuss hand-washing routines. d. Focus on the client's symptoms rather than on the client.

ANS: B Giving information in a calm, simple manner will help the client grasp the important facts. Introducing extraneous topics as described in the distracters will further scatter the client's attention.

A client preparing for surgery has moderate anxiety and is unable to understand preoperative information. Which nursing intervention is most appropriate? a. Reassure the client that all nurses are skilled in providing postoperative care. b. Present the information again in a calm manner using simple language. c. Tell the client that staff is prepared to promote recovery. d. Encourage the client to express feelings to family.

ANS: D Taking an antipsychotic medication coupled with the presence of extrapyramidal symptoms, such as severe muscle stiffness and difficulty swallowing, hyperpyrexia, and autonomic symptoms (pulse elevation), suggest neuroleptic malignant syndrome, a medical emergency. The symptoms given in the scenario are not consistent with the medical problems listed in the incorrect options.

A client receiving risperidone reports severe muscle stiffness at 1030. By 1200, the client has difficulty swallowing and is drooling. By 1600, vital signs are 102.8° F; pulse 110; respirations 26; 150/90. The client is diaphoretic. What is the nurse's best analysis and action? a. Agranulocytosis; institute reverse isolation. b. Tardive dyskinesia; withhold the next dose of medication. c. Cholestatic jaundice; begin a high-protein, high-cholesterol diet. d. Neuroleptic malignant syndrome; notify health care provider stat.

ANS: A "I'm stupid" is a cognitive distortion. A more rational thought is "Sometimes I do stupid things." The latter thinking promotes emotional self-control. The distracters reflect irrational or distorted thinking. This item relates to an audience response question.

A client repeatedly stated, "I'm stupid." Which statement by that client would show progress resulting from cognitive-behavioral therapy? a. "Sometimes I do stupid things." b. "Things always go wrong for me." c. "I always fail when I try new things." d. "I'm disappointed in my lack of ability."

ANS: B Transference refers to feelings a client has toward the health care workers that were originally held toward significant others in his or her life. Countertransference refers to unconscious feelings that the health care worker has toward the client. The superego represents the moral component of personality; it seeks perfection.

A client says to the nurse, "My father has been dead for over 10 years but talking to you is almost as comforting as the talks he and I had when I was a child." Which term applies to the client's comment? a. Superego b. Transference c. Reality testing d. Countertransference

ANS: C Rationalization consists of justifying illogical or unreasonable ideas, actions, or feelings by developing acceptable explanations that satisfy the teller as well as the listener. Denial is an unconscious process that would call for the nurse to ignore the existence of the situation. Projection operates unconsciously and would result in blaming behavior. Compensation would result in the nurse unconsciously attempting to make up for a perceived weakness by emphasizing a strong point.

A client tells a nurse, "My best friend is a perfect person. She is kind, considerate, good-looking, and successful with every task. I could have been like her if I had the opportunities, luck, and money she's had." This client is demonstrating a. denial. b. projection. c. rationalization. d. compensation.

ANS: D Paroxetine is an SSRI and will not produce the same side effects as imipramine, a tricyclic antidepressant. The client will probably not experience dry mouth, constipation, or orthostatic hypotension.

A client tells the nurse, "My doctor prescribed paroxetine for my depression. I assume I'll have side effects like I had when I was taking imipramine." The nurse's reply should be based on the knowledge that paroxetine is included in what class of medication? a. selective norepinephrine reuptake inhibitor (SNRI). b. tricyclic antidepressant. c. monoamine oxidase (MAO) inhibitor. d. selective serotonin reuptake inhibitors (SSRIs)

ANS: B, C, F The Client must avoid hidden sources of alcohol. Many liquid medications, such as cough syrups, contain small amounts of alcohol that could trigger an alcohol-disulfiram reaction. Using alcohol-based skin products such as aftershave or cologne, smelling alcohol-laden fumes, and eating foods prepared with wine, brandy, or beer may also trigger reactions. The other options do not relate to hidden sources of alcohol.

A client undergoing alcohol rehabilitation decides to begin disulfiram therapy. Client teaching should include the need to (Select all that apply.) a. avoid aged cheeses. b. avoid alcohol-based skin products. c. read labels of all liquid medications. d. wear sunscreen and avoid bright sunlight. e. maintain an adequate dietary intake of sodium. f. avoid breathing fumes of paints, stains, and stripping compounds.

ANS: D Denial is an unconscious blocking of threatening or painful information or feelings. Regression involves using behaviors appropriate at an earlier stage of psychosexual development. Displacement shifts feelings to a more neutral person or object. Projection attributes one's own unacceptable thoughts or feelings to another.

A client undergoing diagnostic tests says, "Nothing is wrong with me except a stubborn chest cold." The spouse reports the client smokes, coughs daily, lost 15 pounds, and is easily fatigued. Which defense mechanism is the client using? a. Displacement b. Regression c. Projection d. Denial

ANS: B Situations such as this offer an opportunity to use the client's distractibility to staff's advantage. Clients become frustrated when staff deny requests that the client sees as entirely reasonable. Distracting the client can avoid power struggles. Suggesting that a friend do the shopping would not satisfy the client's need for immediacy and would ultimately result in the extravagant expenditure. Asking whether the client has enough money would likely precipitate an angry response

A client waves a newspaper and says, "I must have my credit card and use the computer right now. A store is having a big sale, and I need to order 10 dresses and four pairs of shoes." What is the nurse's appropriate intervention? a. suggesting the client have a friend do the shopping and bring purchases to the unit. b. inviting the client to sit together and look at new fashion magazines. c. telling the client computer use is not allowed until self-control improves. d. asking whether the client has enough money to pay for the purchases.

ANS: A, D, E Stating the expectation that the client will maintain control of behavior reinforces positive, healthy behavior and avoids challenging the client. Offering as-needed medication provides support for the client trying to maintain control. A firm but calm voice will likely comfort and calm the client. Belittling remarks may lead to aggression. Criticism will probably prompt the client to begin shouting.

A client with a history of command hallucinations approaches the nurse yelling obscenities. Which nursing actions are most likely to be effective in de-escalation for this scenario? (Select all that apply.) a. Stating the expectation that the client will stay in control. b. Asking the client, "Do you want to go into seclusion?" c. Telling the client, "You are behaving inappropriately." d. Offering to provide the client with medication to help. e. Speaking in a firm but calm voice.

ANS: D The client continues to exhibit manic symptoms. Nonadherence to the medication regime is a common problem for clients diagnosed with bipolar disorder. The lithium level should be approaching a therapeutic range after 7 days but may be low from "cheeking" (not swallowing) the medication. The prescribed dose is high, so one would not expect a need for the dose to be increased. Monitoring the client does not address the problem.

A client with diagnosed bipolar disorder was hospitalized 7 days ago and has been taking lithium 600 mg tid. Staff observes increased agitation, pressured speech, poor personal hygiene, and hyperactivity. Which action demonstrates that the nurse understands the most likely cause of the client's behavior? a. Educate the client about the proper ways to perform personal hygiene and coordinate clothing. b. Continue to monitor and document the client's speech patterns and motor activity. c. Ask the health care provider to prescribe an increased dose and frequency of lithium. d. Consider the need to check the lithium level. The client may not be swallowing medications.

ANS: B Emergency seclusion can be affected by a credentialed nurse but must be followed by securing a medical order within a period of time specified by the state and the agency. The incorrect options are not immediately necessary from a legal standpoint. See related audience response question.

A new client acts out so aggressively that seclusion is required before the admission assessment is completed, or orders written. Immediately after safely secluding the client, which action is the nurse's priority? a. Complete the physical assessment. b. Notify the health care provider to obtain a seclusion order. c. Document the incident objectively in the client's medical record. d. Explain to the client that seclusion will be discontinued when self-control is regained.

ANS: C The client with schizotypal personality disorder might have problems thinking, perceiving, and communicating and might have an odd, eccentric appearance; however, they can be made aware of misinterpretations and overtly psychotic symptoms are usually absent. The individual with schizophrenia is more likely to display psychotic symptoms, remain ill for longer periods, and have more frequent and prolonged hospitalizations.

A new psychiatric technician says, "Schizophrenia ... schizotypal! What's the difference?" The nurse's response should include which information? a. A client diagnosed with schizophrenia is not usually overtly psychotic. b. In schizotypal personality disorder, the client remains psychotic much longer. c. With schizotypal personality disorder, the person can be made aware of misinterpretations of reality. d. Schizotypal personality disorder causes more frequent and more prolonged hospitalizations than schizophrenia.

ANS: B Countertransference is the nurse's transference or response to a patient that is based on the nurse's unconscious needs, conflicts, problems, or view of the world. See relationship to audience response question.

A nurse assesses a confused older adult. The nurse experiences sadness and reflects, "This patient is like one of my grandparents ... so helpless." Which response is the nurse demonstrating? a. Transference b. Countertransference c. Catastrophic reaction d. Defensive coping reaction

ANS: C. imipramine Imipramine is a tricyclic antidepressant with strong anticholinergic properties, resulting in dry mouth, blurred vision, constipation, and urinary retention. Lithium therapy is more often associated with fluid-balance problems, including polydipsia, polyuria, and edema. Risperidone therapy is more often associated with movement disorders, orthostatic hypotension, and sedation. Buspirone is associated with anxiety reduction without major side effects.

A nurse can anticipate anticholinergic side effects are likely when a client is prescribed which medication? a. lithium. b. buspirone. c. imipramine. d. risperidone.

ANS: D Schizotypal individuals have poor social skills. Social situations are uncomfortable for them. It is desirable for the individual to develop the ability to meet and socialize with others. Individuals with schizotypal PD (Personality Disorder) usually have no issues with adherence to unit norms, nor are they self-mutilative or manipulative.

A nurse determines desired outcomes for a client diagnosed with schizotypal personality disorder. What is the best outcome? a. The client will adhere willingly to unit norms. b. The client will report decreased incidence of self-mutilative thoughts. c. The client will demonstrate fewer attempts at splitting or manipulating staff. d. The client will demonstrate ability to introduce self to a stranger in a social situation.

ANS: C The DSM-V gives the criteria used to diagnose each mental disorder. It is the official guideline for diagnosing psychiatric disorders. The distracters may not contain diagnostic criteria for a psychiatric illness.

A nurse encounters an unfamiliar psychiatric disorder on a new client's admission form. Which resource should the nurse consult to determine criteria used to establish this diagnosis? a. International Statistical Classification of Diseases and Related Health Problems (ICD-10) b. The ANA's Psychiatric-Mental Health Nursing Scope and Standards of Practice c. Diagnostic and Statistical Manual of Mental Disorders (DSM-V) d. A behavioral health reference manual

ANS: C Families should be made aware that the client will have difficulty concentrating and following or carrying on in-depth or lengthy conversations. The other symptoms and problems are usually seen at later stages of the disease.

A nurse gives anticipatory guidance to the family of a client diagnosed with mild early stage Alzheimer's disease. Which problem common to that stage should the nurse address? a. Violent outbursts b. Emotional disinhibition c. Communication deficits d. Inability to feed or bathe self

A. Report of nausea with frequent episodes of emesis Gastrointestinal upset with nausea and frequent emesis is an early indication of lithium toxicity, therefore, the nurse with hold the prescribe those in obtaining cerium lithium level. The nurse should assess the client for indications of dehydration, which further increases the risk of lithium toxicity. The nurse should expect a weigh gain of up to 2.3 kg during the first week of treatment with lithium. Find ham trimmers are also an expected finding with a client who just started taking lithium. This adverse effect usually diminishes with continued treatment. Serum lithium level of 1.1 mEq/L is within expected reference range.

A nurse is assessing a client prior to administering lithium. The client began taking lithium one week ago for a treatment of mania. For which of the following findings should the nurse withhold dose? A. Report of nausea with frequent episodes of emesis B. Weight gain of 1.8 kg since the start of treatment C. Fine him tremors in both hands D. Serum lithium level of 1.1 mEq/L

A. Tachypnea and tachycardia The greatest race to a client is the development of myocarditis, a potentially fatal adverse effect of clozapine. Myocarditis is an inflammation of the heart muscle that typically occurs within 30 days of starting this medication. Manifestations of myocarditis include chest pain, palpitations, tachycardia, cardiac arrhythmias, dyspnea, tachypnea, a fever, peripheral edema, and unexplained fatigue. Causing pain should be discontinued if the client develops myocarditis and it's used should be avoided in the future. All of the other findings should be reported however this is the priority.

A nurse is assessing a client who began taking clozapine three weeks ago. Which of the following findings should the nurse report to the provider immediately? A. Tachypnea and tachycardia B. Abdominal pain and constipation C. Enuresis and polyuria D. Dry mouth and blurred vision

B. Increase blood pressure Lorazepam is a benzodiazepine that is administered to a client who is experiencing alcohol withdrawal for stabilizing vital signs, preventing seizures and treating delirium tremens. The nurse should anticipate the provider to prescribe lorazepam for increasing blood pressure. Alcohol withdrawal results in an increase pulse rate. Lorazepam does not improve a decrease in urinary output. Reduce urinary output during alcohol withdrawal can be a result of dehydration is treated with oral or intravenous fluids. Lorazepam does not treat nausea. The provider should prescribe an anti-emetic for the client if nausea is presenting during the alcohol withdrawal.

A nurse is assessing a client who is experiencing alcohol withdrawal. For which of the following findings should the nurse anticipate the administration of lorazepam? A. Increased pulse rate B. Increase blood pressure C. Decreased urinary output D. Increased nausea

A. Place the lorazepam on hold Clients who are in a healthcare facility due to an involuntary admission retain the right to refuse treatment, including prescribe medication's. Therefore, the nurse should hold the medication, document the clients wishes in the medical record, and notify the provider of the refusal. Requesting a prescription for an administering I am La Raza plan violates the clients rights and can lead to charges of assault and battery for the nurse. Requesting that another nurse attempt to administer lorazepam also violates the clients rights. Placing the lorazepam in the clients foods also violates the clients rights since they refused the medication.

A nurse is caring for a client has a substance use disorder and was involuntarily admitted by court order for 90 days. When the nurse attempts to administer prescribe oral lorazepam to decrease the manifestations of withdrawal, the client aggressively refuses. Which of the following action should the nurse take? A. Place the lorazepam on hold B. Request a prescription for IM lorazepam C. Request another nurse attempt to administer the lorazepam D. Place the lorazepam in the clients food

a. Aversion therapy

A nurse is caring for a client who has a new prescription for disulfiram for treatment of alcohol use disorder. The nurse informs the client that this medication can cause nausea and vomiting when alcohol is consumed. Which of the following types of treatment is this method an example? a. Aversion therapy b. Flooding c. Biofeedback d. Dialectical behavior therapy

A. Monitor the clients liver function Valproic acid can cause severe hepatotoxicity and liver failure. The nurse should monitor the clients liver function at baseline and periodically there after. The nurse should also teach the client about the manifestations of liver failure. Valproic acid can cause nausea, vomiting, and indigestion that is minimize by taking the medication with food. It does not cause physical dependence or any other significant CNS effects. Consuming foods with tyramine is not an issue with this medication as it would be with a MAOI.

A nurse is caring for a client who has bipolar disorder and a new prescription for valproic acid. Which of the following action should the nurse take? A. Monitor the clients liver function B. Avoid giving the medication with food or milk C. Counsel the client regarding medication dependency D. Limit intake of foods containing tyramine

C. Risperidone Risperidone is a second generation antipsychotic and it is effective in treating NEGATIVE symptoms of schizophrenia such as lack of grooming and flat affect. Chlorpromazine is a first generation antipsychotic and it is used mainly to control positive rather than negative symptoms of schizophrenia. Thiothixene is a first generation antipsychotic and it is used mainly to control positive symptoms of schizophrenia. Haloperidol is a first generation antipsychotic that is also used mainly to control positive symptoms of schizophrenia.

A nurse is caring for a client who has schizophrenia and exhibit a lack of grooming in a flat affect. The nurse should expect a prescription for which of the following medication? A. Chlorpromazine B. Thiothixene C. Risperidone D. Haloperidol

B. My tongue keeps moving like a worm Involuntary tongue movement indicates that this client is at greatest risk for Tardiva dyskinesia, which is a rare neurological syndrome that has no cure. Therefore, this is a priority statement.

A nurse is caring for a client who is receiving chlorpromazine to treat schizophrenia. Which of the following statements by the client should prompt the nurse to notify the provider immediately? A. My last bowel movement was two days ago B. My tongue keeps moving like a worm C. I feel dizzy when I stand up too quickly D. I can't stop blinking when I'm in the sun

C. Reduced aggression Clients who have ADHD can experience a low tolerance for frustration, which can result in aggressive behaviors. Although psychosocial intervention should include developing coping mechanisms in cognitive behavioral therapy, the client might require medication to manage these aggressive behaviors. The nurse should monitor for reduced aggression when a client who has ADHD is taking a mood stabilizer such as lithium. Additional outcomes of mood stabilizing medication's includes decreased impulsivity. Increased attention span would show effectiveness for a stimulant med such as methylphenidate. Decreased anxiety would show effectiveness for a medication like atomoxetine.

A nurse is caring for a client with ADHD who recently started taking lithium. For which of the following findings should the nurse monitor when evaluating the effectiveness of the medication? A. Increased attention span B. Decreased anxiety C. Reduced aggression D. Weight loss

A. Methadone is a replacement for physical dependence to opioids. B. Methadone reduces the unpleasant effects associated with abstinence syndrome. C. Methadone can be used during opioid withdrawal and maintain abstinence. E. Methadone must be prescribed and dispensed by an approved treatment center Methadone is an oral opioid agonist that replaces the opioid to which the client has a physical dependence. The administration prevents abstinence syndrome from occurring. Methadone substitution is used for both opioid withdrawal and long-term maintenance. Due to the risk for a physical dependence, methadone is required to be prescribed and dispensed by an approved treatment center. Disulfiram, rather than methadone, places the client at risk for developing acetaldehyde syndrome if the client consumes alcohol while taking this medication.

A nurse is discussing the use of methadone with a newly licensed nurse. Which of the following statements by the new license nurse indicates an understanding of the teaching? Select all that apply A. Methadone is a replacement for physical dependence to opioids. B. Methadone reduces the unpleasant effects associated with abstinence syndrome. C. Methadone can be used during opioid withdrawal and maintain abstinence. D. Methadone increases the risk for acetaldehyde syndrome. E. Methadone must be prescribed and dispensed by an approved treatment center

D. I am likely to gain weight while taking lithium The nurse should instruct the client to eat a low calorie diet while taking lithium because this medication can cause weight gain. It is encouraged that the client take lithium with food or milk to prevent gastrointestinal upset. The nurse should also instruct the client not to take NSAIDs like ibuprofen with lithium because they increase the risk for lithium toxicity. The nurse should also encourage the client to maintain an adequate dietary intake of sodium. Decreased levels of sodium can result in lithium toxicity.

A nurse is evaluating teaching for a client who has bipolar disorder and a new prescription for lithium. Which of the following statements by the client indicates an understanding of the teaching? A. I should take lithium on an empty stomach B. I can take ibuprofen for headaches while taking lithium C. I need to limit my salt intake while taking lithium D. I am likely to gain weight while taking lithium

c. Bipolar disorder with rapid cycling Indications for ECT include Major depressive disorder if not responsive to pharmacological tx, risks for other tx outweigh ECT, are suicidal or homicidal and need rapid therapeutic response, and those experiencing psychotic manifestations Schizophrenia spectrum disorders with catatonic manifestations or who have schizoaffective disorder Acute manic episodes like bipolar with rapid cycling(4 or more in a year), or unresponsive to tx with lithium and antipsychotic medications

A nurse is leading a peer group discussion about the indications for ECT. Which of the following indications should the nurse include in the discussion? a. Borderline personality disorder b. Acute withdrawal related to a substance use disorder c. Bipolar disorder with rapid cycling d. Dysphoric disorder

B. Naltrexone The nurse should plan to educate the client on the medication naltrexone, an opioid antagonist that is used for long-term maintenance of opioid use disorder. Naltrexone is the usual medication choice following detoxification from opioids.

A nurse is planning care for a client who has completed detoxification from opioid abuse disorder. The nurse should plan to teach about which of the following medications? A. Methadone B. Naltrexone C. Buprenorphine D. Disulfiram

A. Broiled beef steak Phenelzine, and MAOI, is an antidepressant. This medication interacts with a variety of foods to produce a hypertensive crisis. Beef steak and other meats that are fresh do not interact with phenelzine and are safe to consume. Most Jesus, except for cottage cheese and cream cheese interact with MAOIs like phenelzine and can cause hypertensive crisis. Pepperoni, salami, and other dried or cured meats interact with MAIOs. Fish that has been cured or dried interact with MAIOs.

A nurse is planning discharge teaching for a client who has major depressive disorder in a new prescription for phenelzine. Which of the following foods should the nurse include in the plan as safe for the client to consume while taking phenelzine? A. Broiled beef steak B. Macaroni and cheese C. Pepperoni Pizza D. Smoked salmon

C. I can develop lithium toxicity if I experience vomiting or diarrhea Vomiting or diarrhea can cause electrolyte imbalances. If serum sodium decreases, lithium is retained by the kidneys, and the risk for lithium toxicity increases. NSAIDs such as naproxen and ibuprofen increase renal reabsorption of sodium and lithium, which causes an increase in lithium levels and possible toxicity. When sodium levels are low lithium excretion by the kidneys are increase. Therefore eating foods with large amounts of sodium reduces the risk of lithium toxicity. Diuretics decrease kidney excretion of lithium, which can cause lithium levels to rise.

A nurse is providing discharge teaching about lithium toxicity to a client who has a new prescription for lithium. Which of the following statements by the client indicates an understanding of the teaching? A. I should take naproxen if I have a headache because aspirin can cause lithium toxicity B. I can develop lithium toxicity if I eat foods with lots of sodium C. I can develop lithium toxicity if I experience vomiting or diarrhea D. I might need to take a daily diuretic along with my lithium to prevent lithium toxicity

B. I will move slowly when I stand up because amitriptyline can cause my blood pressure to decrease Amitriptyline can cause orthostatic hypotension. The nurse should instruct the client to take precautions for a better injury due to a fall while taking amitriptyline. It also has a sedated effect. This medication often prescribed three times daily until therapeutic dose has been achieved and then entire doses prescribed at bedtime to help the client sleep through the night and prevent daytime drowsiness. Amitriptyline should not be taken with other CNS depressant such as alcohol and sedatives because these substances can enhance adverse effects of it. Amitriptyline and other tricyclic antidepressants have an anticholinergic effects and can cause severe constipation as well as dry mouth, blurred vision, and urinary retention.

A nurse is providing discharge teaching to a client who has been hospitalized for major depressive disorder and has a prescription for amitriptyline. Which of the following statements by the client indicates an understanding of the teaching? A. I will take amitriptyline in the morning because I'll likely have trouble falling asleep if I take it in the evening B. I will move slowly when I stand up because amitriptyline can cause my blood pressure to decrease C. I can drink a glass of beer or wine with my evening meal because amitriptyline does not interact with alcohol D. I will avoid foods high in fiber because amitriptyline causes diarrhea

D. I might not notice the effects of this medication for several weeks The effects of buspirone develop slowly. The initial response takes at least a week, and our peak response takes several weeks. Because a little late action, buspirone should not be taken as PRN medication for the relief of anxiety. Buspirone does not cause dehydration. Buspirone does not have an adverse effect on kidney function. While taking buspirone with food can delay it's absorption, food increases the bioavailability of the medication, allowing it to be more active. Furthermore, taking buspirone with food can lessen the gastrointestinal irritation that it can cause.

A nurse is providing teaching to a client has a new prescription for buspirone. Which of the following statements by the client indicates an understanding of this teaching? A. I need to watch for signs of dehydration B. I need to have my kidney function monitored while taking this medication C. I should take this medication on an empty stomach D. I might not notice the effects of this medication for several weeks

B. This medication takes a few weeks to build up in your system The nurse should inform the client that initial effects of paroxetine take about four weeks to develop. Optimal effects of this medication can be seen in 8 to 12 weeks. This medication is not taken as needed(PRN). The nurse should inform the client that the treatment should continue for at least one year. After that, the provider can prescribe gradual withdrawal if necessary. The nurse should inform the client that withdrawal from paroxetine frequently results in a relapse of the anxiety disorder.

A nurse is providing teaching to a client has social anxiety disorder and a new prescription for a paroxetine. Which of the following statements should the nurse include in the teaching? A. You can take his medication when needed B. This medication takes a few weeks to build up in your system C. You should plan to take this medication for six months D. Relaxing after withdrawing from the medication is rare

C. This medication is an antipsychotic that controls manifestations of schizophrenia Antipsychotic medication's like chlorpromazine are thought to act directly on dopamine receptors in the brain to prevent reuptake of dopamine, thereby controlling psychotic manifestations. Tricyclic antidepressants include amitriptyline and clomipramine. Chlorpromazine is sometimes indicated to control mania and bipolar disorder but it is not an antidepressant. Naltrexone is opiod antagonist used to treat alcohol addiction. Cholinesterase inhibitors that slow progression of dementia are Tacrine and Donepezil.

A nurse is providing teaching to a client who has a new prescription for Chlorpromazine. Which of the following statements should the nurse make? A. This medication is a tricyclic antidepressant and will improve your mood B. This medication is an opioid antagonist that blocks the pleasurable effects of alcohol C. This medication is an antipsychotic that controls manifestations of schizophrenia D. This medication is a cholinesterase inhibitor that slows the progress of dementia

B. I may feel drowsy for weeks after starting this medication. Sedation is an adverse effects of amitriptyline during the first few weeks of therapy. Constipation rather than diarrhea can occur with TCAs, due to the anticholinergic effects. Foods such as pepperoni should be avoided if the client is prescribed an MAOI rather than a TCA like amitriptyline. Observe for manifestations of hypomania or mania caused by CNS stimulation when a client is taking a phenelzine.

A nurse is providing teaching to a client who has a new prescription for amitriptyline. Which of the following statements by the client indicates an understanding of the teaching? A. I can expect to experience diarrhea while taking this medication. B. I may feel drowsy for weeks after starting this medication. C. I cannot eat my favorite pizza with pepperoni while taking this medication. D. This medication will help me lose the weight that I have gained over the last year.

D. Buspirone causes nausea in some people Adverse effects of buspirone include nausea, dizziness, headaches, nervousness, sedation, lightheadedness, and excitement. Buspirone poses a little to no risk of suicidal thoughts. Grapefruit juice increases the levels of buspirone. Elevated levels abuse Perron can cause drowsiness or feelings of disorientation. Buspirone does not enhance a depressant effects of alcohol or barbiturates and other CNS depressants. Buspirone carries no potential for abuse, making it a preferred choice for client to drink alcohol or use drugs.

A nurse is providing teaching to a client who has a new prescription for buspirone to treat anxiety. Which of the following statements should the nurse include in the teaching? A. Use buspirone with caution because it raises the risk for experiencing suicidal thoughts B. You can minimize adverse effects by taking buspirone with grapefruit juice C. Buspirone enhances the depressant effects of alcohol D. Buspirone causes nausea in some people

C. Glucose Clients taking quite a quetiapine are at risk for abnormal glucose metabolism, which can result in diabetes mellitus. Therefore, the client should have a glucose testing periodically. Clients who are taking diuretics would be testing potassium periodically. Clients who have gout would be testing uric acid periodically. Clients who have hypocalcemia would be testing calcium periodically.

A nurse is providing teaching to a client who has schizophrenia and is taking quetiapine fumarate. Which of the following blood test should be performed periodically? A. Potassium B. Uric acid C. Glucose D. Calcium

D. I should expect to feel the full effect on my medication in 2 to 4 weeks The desired response for buspirone can begin within 7 to 10 days, however, the full effect of this medication takes 2 to 4 weeks to occur. Buspirone is an anxiolytic medication that differ significantly from benzodiazepines. This medication does not cause sedation and has no abuse potential. Because the therapeutic effects of buspirone are delayed, it should not be taken on an as needed basis. Buspirone is a nonsedating medication, therefore, the client does not need to take 30 minutes before bedtime. Buspirone should be taken with food to decrease the possibility of nausea. It is usually taken 2 to 3 times daily in divided doses.

A nurse is providing teaching to a client who is generalized anxiety disorder and a new prescription for buspirone. Which of the following statements by the client indicates an understanding of the teaching? A. This medication can cause dependence B. I should take a dose of my medication when I start to feel anxious C. It's important for me to take my medication 30 minutes before bedtime D. I should expect to feel the full effect on my medication in 2 to 4 weeks

A. Elevated blood glucose Finish identify that all second generation antipsychotic medication such as risperidone can cause diabetes, weight gain, and dyslipidemia. To monitor for diabetes, a baseline glucose reading should be obtained and compared to the glucose reading taken 12 weeks later. If there is no change after 12 weeks, glucose should be monitored annually.

A nurse is reviewing lab reports for a client who is taking risperidone. The nurse should identify that which of the following results indicates a potential adverse reaction to the medication? A. Elevated blood glucose B. Elevated WBC count C. Decreased platelet count D. Decreased AST

b. Marriage Rape is adventitious crisis C & D are situational crisis

A nurse is reviewing medical records of multiple clients at a community mental health facility. Which of the following events is an example of client experiencing a maturational crisis? a. Rape b. Marriage c. Severe physical illness d. Job loss

D. Absolute neutrophil count of 1,200 mm^3 The nurse should identify that an absolute neutrophil count of 1200 mm^3 is less than they expected reference range of 2500-8000 mm^3.. An adverse effect of clozapine can include agranulocytosis, which is life-threatening condition in which wbc's including neutrophils are severely decreased. A is WN:, clozapine can causer hyperglycemia. B is WNL. Clozapine can cause dyslipidemia including an elevated triglyceride level. D is WNL

A nurse is reviewing the lab data for a client who is receiving clozapine for schizophrenia. The nurse should identify which of the following findings as an a potential adverse effect of this medication? A. Fasting blood glucose of 95 mg/dL B. Triglycerides 135 mg/dL C. Total cholesterol 175 mg/dL D. Absolute neutrophil count of 1,200 mm^3

C. The client had a motor vehicle crash last year and sustained a head under injury. The greatest risk to a client is the development of seizures. Bupropion can lower the seizure threshold and should be avoided by clients who have a history of a head injury. This information is the highest priority to report to provider.

A nurse is reviewing the medical record of a client who has a new prescription for bupropion for depression. Which of the following findings is the priority for the nurse to report to the provider? A. The client has a family history of seasonal pattern depression. B. The client currently smokes 1.5 packs of cigarettes per day. C. The client had a motor vehicle crash last year and sustained a head under injury. D. The client has a BMI of 25 and has gained 10 pounds over the last year.

C. Valproic acid Valproic acid and lithium are both indicated for the treatment of bipolar disorder. The nurse may safely administer both of these medications to the client. Ibuprofen is not safe to administer to a client who is taking lithium because it can increase kidney absorption of lithium, which can lead to lithium toxicity. Haloperidol is not safe to administer to a client that is taking lithium because the combination increases the clients risk of extrapyramidal adverse effects and tardive dyskinesia. hydrochlorothiazide is not safe to administer to a client who is taking lithium because it promotes sodium loss, which can lead to lithium toxicity.

A nurse is reviewing the medication's of a client who has a bipolar disorder and a new prescription for lithium. Which of the following medication's may be administered safely while the client is taking lithium? A. Ibuprofen B. Haloperidol C. Valproic acid D. Hydrochlorothiazide

b. Administer the medication in the morning c. Monitor for weight loss while taking this medication e. This medication blocks the synaptic reuptake of serotonin in the brain Fluoxetine causes CNS stimulation, which can cause insomnia and why it should be taken in the morning. It can also result in weight loss and it takes about 12 weeks to reach it's full effectiveness. It works by blocking the synaptic reuptake of serotonin in the brain which allows more serotonin to stay at the junction of the neurons.

A nurse is teaching a child who has intermittent explosive disorder about a new prescription for fluoxetine. which of the following information should the nurse provide? SATA a. An adverse effect of this medication is CNS depression b. Administer the medication in the morning c. Monitor for weight loss while taking this medication d. Therapeutic effects of this medication will take 1-3 weeks to fully develop e. This medication blocks the synaptic reuptake of serotonin in the brain

A. Void just before taking this medication C. Wear sunglasses went outside E. Chew sugarless gum Voiding just before taking his medication will help minimize the anticholinergic effects of the urinary hesitancy or retention. Wearing sunglasses when outside will help minimize the anticholinergic effect of photophobia. Chewing sugarless gum will help minimize the anticholinergic effects of dry mouth. Anticholinergic effects do not affect the potassium level of clients and the client SHOULD avoid changing positions quickly to avoid orthostatic hypotension but this is not an anti-cholinergic effect..

A nurse is teaching a client has a new prescription for imipramine how to minimize anticholinergic effects. Which of the following instruction should the nurse include in the teaching? Select all that apply A. Void just before taking this medication B. Increase the dietary intake of potassium C. Wear sunglasses went outside D. Change position slowly when getting up. E. Chew sugarless gum

C. Palpitations

A nurse is teaching a client who has ADHD and is starting therapy with a Amphetamine/dextroamphetamine mixture. Which of the following manifestations to the nurse instruct the client to identify as an adverse effect and report to the provider? A. Restlessness B. Insomnia C. Palpitations D. Weight gain

d. Report confusion as a potential indication of toxicity

A nurse is teaching a client who has a new prescription for alprazolam for generalized anxiety disorder. Which of the following information should the nurse provide? a. Three to six weeks of treatment is required to achieve therapeutic benefit b. Combining alcohol with alprazolam will produce a paradoxical response c. Alprazolam has a lower risk for dependence than other anti anxiety medications d. Report confusion as a potential indication of toxicity

B. I should take this medication before bedtime The nurse should instruct the client that adverse effect of amitriptyline is sedation. The nurse should instruct the client to take the medication at bedtime to minimize sedation during waking hours while promoting sleep. The nurse should instruct the client to take this medication daily as prescribed. The nurse should warn the client not to discontinue this medication abruptly once mood has improved. The nurse should instruct the client that this medication has anticholinergic effects such as dry mouth, blurred vision, urinary retention, and constipation. The nurse should instruct the client that an adverse effect of amitriptyline is weight gain. The nurse should encourage the client to monitor weight routinely during treatment.

A nurse is teaching a client who has a new prescription for amitriptyline to treat depression. Which of the following client statements indicates an understanding of the teaching? A. I should take this medication when I experience active symptoms B. I should take this medication before bedtime C. This medication may cause excess salvation D. I might experience weight loss while taking this medication.

B. Avoid consuming grapefruit juice when taking this medication The nurse should instruct the client to avoid drinking grapefruit juice when taking buspirone because it can cause levels of the medication to increase. Elevated levels can cause drowsiness and subjective effects such as dysphoria. The nurse should inform the client that buspirone has no potential for abuse. The nurse should instruct the client to administer buspirone twice daily. The nurse should instruct the client that peak effects of buspirone takes several weeks to occur.

A nurse is teaching a client who has anxiety and a new prescription for buspirone. Which of the following pieces of information should the nurse include in the teaching? A. Buspirone carries a high potential for abuse B. Avoid consuming grapefruit juice when taking this medication C. Take the medication four times daily D. The peak effects of buspirone occur within one week

B. Chicken salad Phenelzine is an MAOI. Clients taking MAOI must avoid foods that contain tyramine due to the potential for a dangerous food drug interaction. Foods high in tyramine include those that are processed or aged such as luncheon meats and cheeses. This menu selection does not contain food high in tier mean and indicates an understanding of the teaching. Highly processed meat and aged cheeses contains tyramine.

A nurse is teaching an assistive personnel AP about dietary restrictions for a client who is taking phenelzine to treat depression. The AP selection of which of the following foods for the clients lunch indicates an understanding of the teaching? A. Bologna cheese on wheat bread B. Chicken salad C. Cheddar cheese and crackers D. Pizza with pepperoni

B. Drowsiness Drowsiness is an expected side effects of amitriptyline and other tricyclic antidepressants. Sedation is most likely to be present during the first weeks of treatment with amitriptyline and can increase the risk for falls. The nurse should monitor a client who takes amitriptyline for hypotension and EKG changes, such as dysrhythmias. Orthostatic hypotension is a common adverse effect among clients taking amitriptyline, and the nurse should instruct the client to change position slowly when rising from a reclining position. Suicidal thoughts are also something the nurse would monitor for, anxiety and panic attacks are not adverse effects of amitriptyline.

A nurse is updating the plan of care for a client who has a major depression and a new prescription for amitriptyline. The nurse should plan to monitor the client for which of the following adverse effects? A. Hypertension B. Drowsiness C. Panic attacks D. Diarrhea

B. Methadone The nurse anticipate a prescription from the provider for methadone for a client who is experiencing opioid withdrawal. Methadone is an opioid medication that is used for pain management and treatment of withdrawal manifestations and clients who have opioid use disorder. Naloxone is an opioid antagonist that is used to treat opioid overdose. Methylnaltrexone is an opioid antagonist that is used to treat opioid induced constipation for clients who have not responded to other laxatives. Hydromorphone is a strong opioid that is used to treat moderate to severe pain.

A nurse is working in a mental health facility and admitting a client with opioid use disorder who is experiencing withdrawal. The nurse should anticipate a prescription for which of the following medication's from the Rider? A. Methylnaltrexone B. Methadone C. Naloxone D. Hydromorphone

ANS: A, C, D Caffeine is a central nervous system stimulant that acts as an antagonist to the benzodiazepine lorazepam. Daily caffeine intake should be reduced to the amount contained in one cup of coffee. Benzodiazepines are sedatives, thus the importance of exercising caution when driving or using machinery and the importance of not using other central nervous system depressants such as alcohol or sedatives to avoid potentiation. Benzodiazepines do not require a special diet. Food will reduce gastric irritation from the medication.

A nurse plans health teaching for a client diagnosed with generalized anxiety disorder who begins a new prescription for lorazepam. What information should be included? (Select all that apply.) a. Caution in use of machinery b. Foods allowed on a tyramine-free diet c. The importance of caffeine restriction d. Avoidance of alcohol and other sedatives e. Take the medication on an empty stomach

B. Stop the medication administration procedure and say to the patient, "Tell me more about the side effects you've been having." Assess first and the side effects they are having. It is a scheduled medication not an emergency med so you would not force it.

A nurse prepares to administer a scheduled IM injection of an antipsychotic to a patient diagnosed with schizophrenia.. The patient shouts, "Stop! I don't want to take that medicine anymore. I hate the side effects." Select the nurses best action. A. Assemble other staff for a show of force and proceed with the injection, using restraint if necessary. B. Stop the medication administration procedure and say to the patient, "Tell me more about the side effects you've been having." C. Proceed with the injection but explain to the patient that there are medications that will help reduce the unpleasant side effects D. Say to the patient, "Since I've already drawn the medication in the syringe, I'm required to give it, but let's talk to the doctors about delaying next month's dose."

ANS: B The DSM-V details the diagnostic criteria for psychiatric clinical conditions. It is the official guideline for diagnosing psychiatric disorders. The other references are good resources but do not define the diagnostic criteria.

A nurse wants to find a description of diagnostic criteria for anxiety disorders. Which resource would have the most complete information? a. Nursing Outcomes Classification (NOC) b. DSM-V c. The ANA's Psychiatric-Mental Health Nursing Scope and Standards of Practice d. ICD-10

ANS: B A client experiencing severe anxiety has a markedly narrowed perceptual field and difficulty attending to events in the environment. A client experiencing severe anxiety will not learn readily. Determining preferred modes of learning, devising outcomes, and constructing teaching plans are relevant to the task but are not the priority measure. The nurse has already assessed the client's anxiety level. Use of defense mechanisms does not apply.

A nurse wants to teach alternative coping strategies to a client experiencing severe anxiety. Which action should the nurse perform first? a. Verify the client's learning style. b. Lower the client's current anxiety. c. Create outcomes and a teaching plan. d. Assess how the client uses defense mechanisms.

ANS: C Central nervous system (CNS) stimulants are the drugs of choice for treating children diagnosed with ADHD. Methylphenidate and mixed amphetamine salts are most commonly used. None of the other drugs are psychostimulants used to treat ADHD.

A nurse will prepare teaching materials for the parents of a child newly diagnosed with attention deficit hyperactivity disorder (ADHD). Information will focus on which medication likely to be prescribed? a. Paroxetine b. Imipramine c. Methylphenidate d. Carbamazepine

ANS: C Central nervous system (CNS) stimulants are the drugs of choice for treating children diagnosed with ADHD. Methylphenidate and mixed amphetamine salts are most commonly used. None of the other drugs are psychostimulants used to treat ADHD.

A nurse will prepare teaching materials for the parents of a child newly diagnosed with attention deficit hyperactivity disorder (ADHD). Information will focus on which medication likely to be prescribed? a. Paroxetine b. Imipramine c. Methylphenidate d. Carbamazepine

ANS: C Physiological reactions to reminders of the event that include persistent avoidance of stimuli associated with the trauma results in the individual's avoiding talking about the event or avoiding activities, people, or places that arouse memories of the trauma. Avoidance is exemplified by a sense of foreshortened future and estrangement. There is no evidence this soldier is having hyperarousal or reexperiencing war-related traumas. Psychosis is not evident.

A soldier who served in a combat zone returned to the United States. The soldier's spouse complains to the nurse, "We had planned to start a family, but now he won't talk about it. He won't even look at children." The spouse is describing which symptom associated with post- traumatic stress disorder (PTSD)? a. Re-experiencing b. Hyperarousal c. Avoidance d. Psychosis

ANS: B Countertransference occurs when the nurse unconsciously and inappropriately displaces onto the patient feelings and behaviors related to significant figures in the nurse's past. In this instance, the new nurse's irritability stems from relationships with parents. The distracters indicate transference or accurate analysis of the patient's behavior.

An advanced practice nurse observes a novice nurse expressing irritability regarding a patient with a long history of alcoholism and suspects the new nurse is experiencing countertransference. Which comment by the new nurse confirms this suspicion? a. "This patient continues to deny problems resulting from drinking." b. "My parents were alcoholics and often neglected our family." c. "The patient cannot identify any goals for improvement." d. "The patient said I have many traits like her mother."

ANS: A Rationalization involves unconsciously making excuses for one's behavior, inadequacies, or feelings. Regression involves the unconscious use of a behavior from an earlier stage of emotional development. Compensation involves making up for deficits in one area by excelling in another area. Introjection is an unconscious, intense identification with another person.

An individual experiences sexual dysfunction and blames it on a partner by calling the person unattractive and unromantic. Which defense mechanism is evident? a. Rationalization b. Compensation c. Introjection d. Regression

ANS: A Delirium is characterized by an abrupt onset of fluctuating levels of awareness, clouded consciousness, perceptual disturbances, and disturbed memory and orientation. The onset of dementia or Alzheimer's disease, a type of dementia, is more insidious. Amnestic syndrome involves memory impairment without other cognitive problems.

An older adult client takes multiple medications daily. Over 2 days, the client developed confusion, slurred speech, an unsteady gait, and fluctuating levels of orientation. What are these findings most characteristic of? a. delirium. b. dementia. c. amnestic syndrome. d. Alzheimer's disease.

ANS: C The individual with a borderline personality disorder characteristically demonstrates manipulative, splitting, and self-destructive behaviors. Consistent limit setting is vital not only for the client's safety, but also to prevent splitting other staff. "Why" questions are not therapeutic.

As a nurse prepares to administer medication to a client diagnosed with a borderline personality disorder, the client says, "Just leave it on the table. I'll take it when I finish combing my hair." What is the nurse's best response? a. Reinforce this assertive action by the client. Leave the medication on the table as requested. b. Respond to the client, "I'm worried that you might not take it. I'll come back later." c. Say to the client, "I must watch you take the medication. Please take it now." d. Ask the client, "Why don't you want to take your medication now?"

ANS: C Partial hospitalization will provide services the client needs as well as give supervision and meals to the client while the daughter is at work. Home care would not provide socialization. The client does not need the intensity of a skilled nursing facility. A halfway house provides 24-hour care and usually expects involvement in off-campus programs.

Discharge planning begins for an elderly client hospitalized for 2 weeks diagnosed with major depressive disorder. The client needs ongoing assessment and socialization opportunities as well as education about medication and relapse prevention. The client lives with a daughter, who works during the week. What is the best referral for this client? a. Behavioral health home care b. A skilled nursing facility c. Partial hospitalization d. A halfway house

ANS: B Confabulation refers to making up of stories or answers to questions by a person who does not remember. It is a defensive tactic to protect self-esteem and prevent others from noticing memory loss. The client's response was not sundown syndrome. Perseveration refers to repeating a word or phrase over and over. Delirium is not present in this scenario.

During morning care, a nurse asks a client diagnosed with dementia, "How was your night?" The client replies, "It was lovely. I went out to dinner and a movie with my friend." Which term applies to the client's response? a. Sundown syndrome b. Confabulation c. Perseveration d. Delirium

ANS: D Calm, brief, directive verbal interaction can help the client gain control of overwhelming feelings and impulses related to anxiety. Clients experiencing panic-level anxiety are unable to focus on reality; thus, learning relaxation techniques is virtually impossible. Administering anxiolytic medication should be considered if providing calm, brief, directive communication is ineffective. Although the client is disorganized, violence may not be imminent, ruling out the intervention of preparing for physical control until other less-restrictive measures are proven ineffective

For a client experiencing panic, which nursing intervention should be implemented first? a. Teach relaxation techniques. b. Administer an anxiolytic medication. c. Prepare to implement physical controls. d. Provide calm, brief, directive communication. .

ANS: D This is a manipulative behavior. Because manipulation violates the rights of others, limit setting is absolutely necessary. Furthermore, limit setting is necessary in this case because the safety of at least two other clients is at risk. Limit setting may occasionally be used with dependent behavior (clinging to the nurse) and histrionic behavior (flirting with staff members), but other therapeutic techniques are also useful. Limit setting is not needed for a client who is hypervigilant and refuses to attend unit activities; rather, the need to develop trust is central to client compliance.

For which client behavior would limit setting be most essential? a. The client who clings to the nurse and asks for advice about inconsequential matters. b. The client who is flirtatious and provocative with staff members of the opposite sex. c. The client who is hypervigilant and refuses to attend unit activities. d. The client who urges a suspicious client to hit anyone who stares.

C. The client reports eating excessively When amphetamine is taken at a therapeutic dose, it causes appetite suppression. Abrupt withdrawal of amphetamine can result in abstinence syndrome in a client who is physically dependent on the medication. Indications of physical dependence include excessive eating, exhaustion, depression, prolong sleep, enter craving for more amphetamine.

I nurse is assessing a client who has ADHD and reports abruptly discontinuing his amphetamine treatment. Which of the following assessments indicate that the client is physically dependent on amphetamines? A. The client exhibits paranoia B. The client reports having insomnia C. The client reports eating excessively D. The client has an increase heart rate

D. The DSM-V diagnosis guides medical treatment, whereas a nursing diagnosis offers a framework for identifying interventions for the issues a patient is experiencing

Select the best response for the nurse to a question from another health professional regarding the difference between a diagnosis and DSM-V in a nursing diagnosis. A. There is no functional difference between the two. Both identify human disorders. B. The DSM-V diagnosis disregards culture, whereas the nursing diagnosis takes culture into account. C. The DSM-V diagnosis describes causes of disorders where as a nursing diagnosis does not explore etiology. D. The DSM-V diagnosis guides medical treatment, whereas a nursing diagnosis offers a framework for identifying interventions for the issues a patient is experiencing.

A. Anhedonia and feelings of worthlessness Anhedonia are not able to feel pleasure The other answers are related to maniac episodes

The client is admitted for major depression what does a nurse expect to find during the assessment? A. Anhedonia and feelings of worthlessness B. Depressed mood and pressured speech C. Changes in sleep pattern and grandiose mood D. Difficulty focusing and flight of ideas

ANS: A, B, C, E, F These assessment findings are consistent with the symptoms of PTSD. Ritualistic behaviors are expected in obsessive-compulsive disorder.

The nurse interviewing a client with suspected post-traumatic stress disorder (PTSD) should be alert to which client findings? (Select all that apply.) a. avoids people and places that arouse painful memories. b. experiences flashbacks or re-experiences the trauma. c. experiences symptoms suggestive of a heart attack. d. feels compelled to repeat selected ritualistic behaviors. e. demonstrates hypervigilance or distrusts others. f. feels detached, estranged, or empty inside.

B. Monitoring vital signs in cardiac rhythm

The nurse is caring for a client who just received ECT treatment. What is the priority nursing intervention? A. Nutrition and hydration B. Monitoring vital signs in cardiac rhythm C. Reducing disorientation and confusion D. Assisting the client to identify negative thoughts

A. "What do you know about Zyprexa Relprev?" Fluoxetine is not used to treat schizophrenia...zyprexa is an antipsychotic

The nurse is caring for a client with schizophrenia who is hospitalized for the fourth time in six months due to non-adherence of his medication. The client states, "I just can't remember to take my medication." Which reply is most appropriate? A. "What do you know about Zyprexa Relprev?" B. "If it continues you will be court committed." C. "When you miss a does, just take it the next time you remember." D. "Have you considered a long acting form of Fluoxetine?"

d. Buspirone Buspirone is used to treat anxiety disorders or in the short-term treatment of symptoms of anxiety. Buspirone has been shown to be as effective as diazepam and clorazepate and more effective than placebo in the treatment of generalized anxiety. IT IS NOT A CONTROLLED SUBSTANCE, not habit forming and not a PRN. Buspirone lacks the sedative, muscle relaxant, and anticonvulsive effects of the benzodiazepines. Quetiapine is used for schizophrenia, acute manic episodes, and adjunctive treatment for major depressive disorder. Omeprazole is used for heartburn and indigestion Lorazepam is a benzo

The nurse is caring for a patient frequently experiencing moderate levels of anxiety and determines the patient would benefit from an atypical (nonbenzoid) anxiolytic. The nurse prepares a SBAR and recommends which medication? a. Lorazepam b. Quetiapine c. Omeprazole d. Buspirone

C. Ensure that the patient's nutritional needs are met

The nurse is developing a plan of care for a patient in the manic stage of bipolar disorder. What intervention should be included? A. Touch the patient to provide reassurance B. Invite the patient to lead a community meeting C. Ensure that the patient's nutritional needs are met D. Design activities that require the patient's concentration

ANS: D This client could profit from the structure and supervision provided by spending the day at the partial hospitalization program. During the evening, at night, and on weekends, the spouse could assume responsibility for supervision. A suicidal client needs inpatient hospitalization. The other clients can be served in the community or with individual visits.

The nurse should refer which of the following clients to a partial hospitalization program? a. One who has a therapeutic lithium level and reports regularly for blood tests and clinic follow-up. b. One who needs psychoeducation for relaxation therapy related to agoraphobia and panic episodes. c. One who spent yesterday in a supervised crisis care center and continues to have active suicidal ideation. d. One who cannot avoid using alcohol when their spouse goes to work every morning.

ANS: D The correct answer is the most global response. Cultural competence requires ongoing effort. Culture is dynamic, diversified, and changing. The nurse must be prepared to gain cultural knowledge and determine nursing care measures that clients find acceptable and helpful. Interpreting the thinking of individual clients does not ensure culturally competent care. Reducing a client's ethnocentrism may not be a desired outcome.

To provide culturally competent care, the nurse should focus on what action? a. accurately interpreting the thinking of individual clients. b. predicting how a client may perceive treatment interventions. c. formulating interventions to reduce the client's ethnocentrism. d. identifying strategies that fit within the cultural context of the client.

ANS: B Clients have a right to treatment in the least restrictive setting. Safety is important, but less restrictive measures should be tried first. Unnecessary seclusion may result in a charge of false imprisonment. Seclusion violates the client's autonomy. The principle by which the nurse is motivated is beneficence, not justice. The tort represented is false imprisonment.

Two hospitalized clients fight whenever they are together. During a team meeting, a nurse asserts that safety is of paramount importance, so treatment plans should call for both clients to be secluded to keep them from injuring each other. What would be the outcome of this assertion? a. reinforcement of the autonomy of the two clients. b. violation of the civil rights of both clients. c. commission of an intentional tort of battery. d. Correct placement on emphasis on safety.

ANS: C Projection is the hallmark of blaming, scapegoating, prejudicial thinking, and stigmatizing others. Conversion involves the unconscious transformation of anxiety into a physical symptom. Introjection involves intense, unconscious identification with another person. Splitting is the inability to integrate the positive and negative qualities of oneself or others into a cohesive image.

Two staff nurses applied for a charge nurse position. After the promotion was announced, the nurse who was not promoted said, "The nurse manager had a headache the day I was interviewed." Which defense mechanism is evident? a. Introjection b. Conversion c. Projection d. Splitting

ANS: C This is an early outcome that paves the way for later taking greater responsibility for controlling manipulative behavior. Identifying anger relates to anger and aggression control. Using manipulation to get legitimate needs is an inappropriate outcome. The client would ideally use assertive behavior to promote need fulfillment. Accepting fulfillment of requests within an hour rather than immediately relates to impulsivity control.

What is an appropriate initial outcome for a client diagnosed with a personality disorder who frequently manipulates others? a. The client will identify when feeling angry. b. The client will use manipulation only to get legitimate needs met. c. The client will acknowledge manipulative behavior when it is called to his or her attention. d. The client will accept fulfillment of his or her requests within an hour rather than immediately.

ANS: B One goal of health teaching is normalization of eating habits. Food restriction and skipping meals lead to rebound bingeing. Teaching the client to eat a small meal after purging will probably perpetuate the need to induce vomiting. Teaching the client to eat a large breakfast but no lunch and increase intake after 4 PM will lead to late-day bingeing. Journal entries are private.

What is an appropriate intervention for a client diagnosed with bulimia nervosa who binges, and purges is to teach the client? a. to eat a small meal after purging. b. not to skip meals or restrict food. c. to increase oral intake after 4 PM daily. d. the value of reading journal entries aloud to others.

ANS: B Maintaining consistent limits is by far the most difficult intervention because of the client's superior skills at manipulation. Supporting behavioral change and monitoring client safety are less difficult tasks. Aversive therapy would probably not be part of the care plan because positive reinforcement strategies for acceptable behavior seem to be more effective than aversive techniques.

What is the most challenging nursing intervention with clients diagnosed with personality disorders who use manipulation? a. Supporting behavioral change b. Maintaining consistent limits c. Monitoring suicide attempts d. Using aversive therapy

ANS: C The most common side effects are gastrointestinal disturbances, reduced appetite, weight loss, urinary retention, dizziness, fatigue, and insomnia. Weight loss has the potential to interfere with the child's growth and development. The distracters relate to side effects of conventional antipsychotic medications

What is the nurse's priority focused assessment for side effects in a child taking methylphenidate for attention deficit hyperactivity disorder (ADHD)? a. Dystonia, akinesia, and extrapyramidal symptoms b. Bradycardia and hypotensive episodes c. Sleep disturbances and weight loss d. Neuroleptic malignant syndrome

ANS: B Careful observation and supervision are of ultimate importance because an appropriate outcome would be that the client will remain safe and free from injury. Physical contact during care cannot be avoided. Activating a bed alarm is only one aspect of providing for the client's safety.

What is the priority intervention for a client diagnosed with delirium who has fluctuating levels of consciousness, disturbed orientation, and perceptual alterations? a. Distraction using sensory stimulation b. Careful observation and supervision c. Avoidance of physical contact d. Activation of the bed alarm

ANS: A Clients with schizotypal personality disorder are eccentric and often display perceptual and cognitive distortions. They are suspicious of others and have considerable difficulty trusting. They become highly anxious and frightened in social situations, thus the need to respect their desire for social isolation. Teaching the client to match clothing is not the priority intervention. Clients with schizotypal personality disorder rarely engage in behaviors that violate the nurse's rights or exploit the nurse.

What is the priority intervention for a nurse beginning to work with a client diagnosed with a schizotypal personality disorder? a. Respect the client's need for periods of social isolation. b. Prevent the client from violating the nurse's rights. c. Teach the client how to select clothing for outings. d. Engage the client in community activities.

ANS: A An overdose of stimulants, such as amphetamines, can produce respiratory and circulatory dysfunction as well as hyperthermia. Concentration is impaired. This client will be hypervigilant; it is not necessary to awaken the client.

What is the priority nursing intervention when caring for a client after an overdose of amphetamines? a. Monitor vital signs. b. Observe for depression. c. Awaken the client every 15 minutes. d. Use warmers to maintain body temperature.

ANS: A Typical antipsychotic drugs often produce sedation and extrapyramidal side effects such as stiffness and gait disturbance, effects the client might describe as making him or her feel like a "robot." The side effects mentioned in the other options are usually not associated with typical antipsychotic therapy or would not have the effect described by the client.

When a client diagnosed with schizophrenia was discharged 6 months ago, haloperidol was prescribed. The client now says, "I stopped taking those pills. They made me feel like a robot." What are common side effects the nurse should validate with the client? a. Sedation and muscle stiffness b. Sweating, nausea, and diarrhea c. Mild fever, sore throat, and skin rash d. Headache, watery eyes, and runny nose

ANS: A ODD is a repeated and persistent pattern of having an angry and irritable mood in conjunction with demonstrating defiant and vindictive behavior. The distracters identify findings associated with conduct disorder (CD), anxiety disorder, and Tourette's syndrome

Which assessment findings support a diagnosis of oppositional defiant disorder (ODD)? a. Negative, hostile, and spiteful toward parents. Blames others for misbehavior. b. Exhibits involuntary facial twitching and blinking; makes barking sounds. c. Violates others' rights; cruelty toward people or animals; steals; truancy. d. Displays poor academic performance and reports frequent nightmares.

ANS: A, B, C Disorientation to place and time is an expected finding. Orientation to person (self) usually remains intact. Attention span is short, and difficulty focusing or shifting attention as directed is often noted. Clients with delirium commonly experience illusions and hallucinations. Fluctuating levels of consciousness are expected. Agnosia occurs with dementia. Apathy is associated with depression.

Which assessment findings would the nurse expect in a client experiencing delirium? (Select all that apply.) a. Impaired level of consciousness b. Disorientation to place, time c. Wandering attention d. Apathy e. Agnosia

ANS: C These foods provide adequate nutrition, but more importantly, they are finger foods that the hyperactive client could eat while in motion. The foods in the incorrect options cannot be eaten without utensils

Which dinner menu is best suited for a client with acute mania? a. Spaghetti and meatballs, salad, and a banana b. Beef and vegetable stew, a roll, and chocolate pudding c. Broiled chicken breast on a roll, an ear of corn, and an apple d. Chicken casserole, green beans, and flavored gelatin with whipped cream

ANS: A The descriptors given indicate the client is functioning at an optimal level, using appropriate behavior, and thinking without becoming overstimulated by unit activities. The incorrect options reflect manic behavior.

Which documentation indicates that the treatment plan for a client diagnosed with acute mania has been effective? a. "Converses with few interruptions; clothing matches; participates in activities." b. "Irritable, suggestible, distractible; napped for 10 minutes in afternoon." c. "Attention span short; writing copious notes; intrudes in conversations." d. "Heavy makeup; seductive toward staff; pressured speech."

D. Throws a heavy plate at a waiter at the direction of command hallucinations

Which individual diagnosed with mental illness may need emergency or involuntary emission? The individual who... A. Resumes using heroin while still taking naltrexone (ReVia) B. Reports hearing angels playing harps during thunderstorms C. Does not keep an outpatient appointment with the mental health nurse D. Throws a heavy plate at a waiter at the direction of command hallucinations

ANS: B Clozapine therapy may produce neutropenia; therefore, signs of infection should be immediately reported to the health care provider. In addition, the client should have white blood cell levels measured weekly. The other options are not relevant to clozapine.

Which instruction has priority when teaching a client about clozapine? a. "Avoid unprotected sex." b. "Report sore throat and fever immediately." c. "Reduce foods high in polyunsaturated fats." d. "Use over-the-counter preparations for rashes."

ANS: A Manipulative people frequently make requests of many different staff, hoping one will give in. Having one decision maker provides consistency and avoids the potential for playing one staff member against another. Positive reinforcement of appropriate behaviors is more effective than negative reinforcement. The behavior should not be ignored; judicious use of confrontation is necessary. Clients with antisocial personality disorders rarely have feelings of fear and inferiority.

Which intervention is appropriate for an individual diagnosed with an antisocial personality disorder who frequently manipulates others? a. Refer requests and questions related to care to the case manager. b. Encourage the client to discuss feelings of fear and inferiority. c. Provide negative reinforcement for acting-out behavior. d. Ignore, rather than confront, inappropriate behavior.

ANS: D Naltrexone is useful for treating both opioid and alcohol addiction. An opioid antagonist blocks the action of opioids and the mechanism of reinforcement. It also reduces or eliminates alcohol craving. None of the other options are associated with such a response.

Which medication to maintain abstinence would most likely be prescribed for clients diagnosed with an addiction to either alcohol or opioids? a. Bromocriptine b. Methadone c. Disulfiram d. Naltrexone

ANS: C, D, E The correct answers are consistent with problems frequently identified for clients with late-stage Alzheimer's disease. Confusion is chronic, not acute. The client's cognition is too impaired to grieve.

Which nursing diagnoses are most applicable for a client diagnosed with severe late stage Alzheimer's disease? (Select all that apply.) a. Acute confusion b. Anticipatory grieving c. Urinary incontinence d. Disturbed sleep pattern e. Risk for caregiver role strain

ANS: B False imprisonment involves holding a competent person against his or her will. Actual force is not a requirement of false imprisonment. The individual needs only to be placed in fear of imprisonment by someone who has the ability to carry out the threat. If a client is not competent (confused), then the nurse should act with beneficence. Clients admitted involuntarily should not be allowed to leave without permission of the treatment team.

Which nursing intervention demonstrates false imprisonment? a. A confused and combative client says, "I'm getting out of here, and no one can stop me." The nurse restrains this client without a health care provider's order and then promptly obtains an order. b. A client has been irritating and attention seeking much of the day. A nurse escorts the client down the hall saying, "Stay in your room, or you'll be put in seclusion." c. An involuntarily hospitalized client with suicidal ideation runs out of the psychiatric unit. The nurse rushes after the client and convinces the client to return to the unit. d. An involuntarily hospitalized client with homicidal ideation attempts to leave the facility. A nurse calls the security team and uses established protocols to prevent the client from leaving.

ANS: C Olanzapine is a short-acting antipsychotic useful in calming angry, aggressive clients regardless of diagnosis. The other drugs listed require long-term use to reduce anger. Lithium is for bipolar clients. Trazodone is commonly prescribed for clients experiencing depression, insomnia, or chronic pain. Valproic acid is for bipolar or borderline clients.

Which prescribed medication should a nurse administer to provide immediate intervention for a psychotic client whose aggressive behavior continues to escalate despite verbal intervention? a. Lithium b. Trazodone c. Olanzapine d. Valproic acid

ANS: C Seeking a staff member instead of impulsively self-mutilating shows an adaptive coping strategy. The incorrect responses demonstrate idealization, devaluation, and wishful thinking.

Which statement made by a client diagnosed with borderline personality disorder indicates the treatment plan is effective? a. "I think you are the best nurse on the unit." b. "I'm never going to get high on drugs again." c. "I felt empty and wanted to hurt myself, so I called you." d. "I hate my mother. I called her today, and she wasn't home."

ANS: A, B, D, E A client with hypomania is expansive, grandiose, and labile; uses poor judgment; spends inappropriately; and is over-stimulated by a busy environment. Providing structure helps the client maintain appropriate behavior. Financial irresponsibility may be avoided by limiting access to cash and credit cards. Continued decline in sleep patterns may indicate the condition has evolved to full mania. Group socialization should be kept to a minimum to reduce stimulation. The family should supervise medication administration to prevent deterioration to a full manic episode and because the client is at risk to omit medications.

Which suggestions are appropriate for the family of a client diagnosed with bipolar disorder who is being treated as an outpatient during a hypomanic episode? (Select all that apply.) a. Limit credit card access. b. Provide a structured environment. c. Encourage group social interaction. d. Supervise medication administration. e. Monitor the client's sleep patterns.

A. Punishment Aversion therapy is a type of behavioral therapy. Think of punishment to decrease undesired behavior. Example slapping a hand every time someone smokes. Another example, Disulfiram makes people sick if they drink alcohol. An example of systematic desensitization is often used with phobias, user exposure therapy to make them used to it, a little by little. Role modeling is not a behavioral therapy it is psychodynamic. Positive reinforcement is the opposite of aversion therapy.

Which technique is most applicable to aversion therapy? A. Punishment B. Desensitization C. Role modeling D. Positive reinforcement

ANS: A Restating allows the client to validate the nurse's understanding of what has been communicated. Restating is an active listening technique. Judgments should be suspended in a nurse-client relationship. Close-ended questions such as "Did you feel angry?" ask for specific information rather than showing understanding. When the nurse simply states that he or she understands the client's words, the client has no way of measuring the understanding.

Which technique will best communicate to a client that the nurse is interested in listening? a. Restating a feeling or thought the client has expressed. b. Asking a direct question, such as "Did you feel angry?" c. Making a judgment about the client's problem. d. Saying, "I understand what you're saying."


Related study sets

Chapter 10 Section 2 Quiz Ghana (West Africa)

View Set

The soothing power of books - 6 Minute English

View Set

Hydrogeology (Lecture 7) Hydrogeological Properties

View Set

Introductory Physics 1 - Test 3 (Torque & Equilibrium)

View Set

Algebra 2 - Chapter 4 (Systems of Equations and Inequalities)

View Set

Ch. 12 one way between subjects anova

View Set

Ch 7: Observational and Implicit Learning, and Learning in the Classroom

View Set